Elementary number theory homework 1-15

本文最后更新于:8 个月前

作业 \(\mathbf{PDF}\) 文件下载请点击这里 初等数论作业

\(\large\textcolor{blue}{初等数论第一次作业}\ \ \ \ \ \ _\textcolor{blue}{2022.2.22}\)

习题 1.1.1

\(n\) 是奇数, 则 \(8 \mid n^{2}-1\).


证明:由 \(n\) 是奇数,则令 \(n=2k+1\),有 \(n^2-1=(2k+1)^2-1=4k(k+1)\),由于 \(k\)\(k+1\) 一奇一偶,从而 \(2\mid k(k+1)\),则 \(8\mid n^2-1\)

习题1.1.2

\(n \geqslant 3\) 是奇数, 证明: \(\left(1+\dfrac{1}{2}+\dfrac{1}{3}+\cdots+\dfrac{1}{n-1}\right)(n-1) !\)\(n\) 整除.


证明:由 \(n\in \mbox{odd}\) 则可以依次首尾相加得到 \[ 1+\dfrac{1}{2}+\cdots+\dfrac{1}{n-1}=(1+\dfrac{1}{n-1})+(\dfrac{1}{2}+\dfrac{1}{n-2})+\cdots+(\dfrac{1}{\dfrac{n-1}{2}}+\dfrac{1}{\dfrac{n+1}{2}})\\ =n\cdot(\dfrac{1}{1\cdot(n-1)}+\dfrac{1}{2\cdot(n-2)}+\cdots+\dfrac{1}{\dfrac{n-1}{2}\cdot\dfrac{n+1}{2}}) \]\((n-1)!\) 均有 \(1,2,\cdots,n-1\) 的因子,从而 \(n\mid (1+\dfrac{1}{2}+\cdots+\dfrac{1}{n-1})(n-1)!\)

习题1.1.3

\(m\)\(n\) 为正整数, \(m \geqslant 3\). 证明: \(2^{m}-1 \not\mid 2^{n}+1\).


引理 \(1.1.1\) \(q\in Z,x-1\mid x^{q}-1\),这是由于 \(1+x+\cdots+x^{q-1}=\dfrac{x^{q}-1}{x-1}\in Z\)

证明:显然当 \(m>n\) 时结论不成立,假设结论不成立 \(2^{m}-1\mid 2^{n}+1\),由带余除法定理有 \(n=mq+r,r<m\) \[ \because 2^{n}+1=2^{mq+r}+1=2^{r}\cdot (2^{mq}-1)+2^{r}+1,2^{m}-1\mid 2^{mq}-1\\ \therefore 2^{m}-1\mid 2^{r}+1\ 而\ r<m,2^{m}-1>2^{r}+1\Longleftrightarrow 2^{m}>2^{r}+2\Longleftrightarrow 2^{m}-2^{r}>2^{m-1}\geq4>2 \] 从而假设不成立,\(2^{m}-1\nmid 2^{n}+1\)

习题1.1.4

\(q\) 是大于 1 的整数. 证明:

\((i)\) 每个正整数 \(n\) 可以惟一地表示成 \[ n=a_{0}+a_{1} q+a_{2} q^{2}+\cdots+a_{k} q^{k}, \] 其中 \(a_{i}\) 是满足 \(0 \leqslant a_{i} \leqslant q-1\) 的整数 \((0 \leqslant i \leqslant k)\), 并且 \(a_{k} \neq\) 0. 这叫作 \(n\)\(q\) 进制表示.

\((ii)\) \(a_{i}=\left[\dfrac{n}{q^{i}}\right]-q\left[\dfrac{n}{q^{i+1}}\right](0 \leqslant i \leqslant k)\).


证明:\((i)\) 带余除法定理令 \(n=b_0+m_1q,0\leq b_0\leq q-1\),对 \(m_i\) 重复上述步骤有 \[ m_1=b_1+m_2q,0\leq b_1\leq q-1\cdots ,m_k=b_k,0\leq b_k\leq q-1 \] 则回代得到 \(n=b_0+q(b_1+q(b_2+\cdots+(b_k)))\),展开得到 \(n=a_0+a_1q+\cdots+a_kq^k\)

唯一性的证明:若有两种表示方法 \(n=a_0+a_1q+\cdots+a_kq^k=c_0+c_1q+\cdots+c_kq^k\),有 \(q\mid a_0-c_0\),则由于 \(0\leq a_0,c_0\leq q-1\)\(a_0=c_0\),代入消去有 \(a_1+a_2q+\cdots+a_kq^k=c_1+c_2q+\cdots+c_kq^k\),同理有 \(a_i=c_i\),则表示方法唯一

\((ii)\)\((i)\) 中表示有 \(n=a_0+a_1q+\cdots+a_kq^k\),两边除以 \(q^{i}\)\[ \dfrac{n}{q^{i}}=\dfrac{a_0}{q^{i}}+\dfrac{a_1}{q^{i-1}}+\cdots a_i+a_{i+1}q+a_{i+2}q^2+\cdots+a_{k}q^{k-i} \]\(0\leq a_i\leq q-1\) 且有取整函数的定义可知 \([\dfrac{n}{q^{i}}]=a_i+a_{i+1}q+a_{i+2}q^2+\cdots+a_{k}q^{k-i}\)

除以 \(q^{i+1}\) 重复上述步骤得 \([\dfrac{n}{q^{i+1}}]=a_{i+1}+a_{i+2}q+\cdots+a_{k}q^{k-i-1}\),比对上述两式得 \(a_i=[\dfrac{n}{q^{i}}]-q[\dfrac{n}{q^{i+1}}]\)

习题1.1.5

\(\alpha_{1}, \cdots, \alpha_{n}\) 为实数 \((n \geqslant 2)\), 证明: \[ \begin{aligned} {\left[\alpha_{1}\right]+\left[\alpha_{2}\right]+\cdots+\left[\alpha_{n}\right] } & \leqslant\left[\alpha_{1}+\alpha_{2}+\cdots+\alpha_{n}\right] \\ & \leqslant\left[\alpha_{1}\right]+\left[\alpha_{2}\right]+\cdots+\left[\alpha_{n}\right]+n-1 \end{aligned} \]


证明:记 \(\alpha_i=[\alpha_i]+\{\alpha_i\},\forall \ x\in R,\{x\}\in[0,1)\),从而有 \(\displaystyle [\sum_{i=1}^n\alpha_i]=\sum_{i=1}^n[\alpha_i]+[\sum_{i=1}^{n}\{\alpha_i\}]\geq\sum_{i=1}^n[\alpha_i]\)

\(\displaystyle [\sum_{i=1}^{n}\{\alpha_i\}]<[\sum_{i=1}^{n}1]=n\),由取整特性 \(\displaystyle [\sum_{i=1}^{n}\{\alpha_i\}]\leq n-1\),则 \(\displaystyle\sum_{i=1}^n[\alpha_i]\leq [\sum_{i=1}^n\alpha_i]\leq \displaystyle\sum_{i=1}^n[\alpha_i]+n-1\)

习题1.1.11

\(n\) 为正整数, \(n \geqslant 2\). 如果 \(n\) 没有小于或等于 \(\sqrt{n}\) 的素数因子, 则 \(n\) 为素数.


证明:反证法,若 \(n\) 为合数,则 \(n\) 有素因子 \(q\),则 \(n=mq,m,q\in Z\),由于 \(n\) 没有 \(\leq\sqrt{n}\) 的素数因子 \[ \therefore m,q>\sqrt{n},mq>n \] 与假设矛盾,从而 \(n\) 为素数

习题1.1.12

对每个整数 \(n \geqslant 3, n\)\(n !\) 之间必有素数. 由此证明素数有无限多个.


证明:反证法,假设 \(n\)\(n!\) 之间全为合数,则 \(n!-1\) 为合数,其一定有素数因子,由假设知 \(n+1,n+2,\cdots,n!\) 全为合数,不可能是 \(n!-1\) 的素数因子,另一方面,\(\forall \ n\geq a\geq 2,a\mid n!\),从而 \[ P=\{p\mid p\in[2,n]\ \cap\ \mbox{p is prime} \},\forall \ x\in P,x\nmid n!-1 \]\(n!-1\) 没有小于其自身的素数因子,从而 \(n!-1\) 为素数,与假设矛盾,从而 \(n\)\(n!\) 之间一定有素数

\(3\) 开始, \((3,3!)\) 之间有素数,\((3!,(3!)!)\) 之间有素数,进而构造数列 \[ a_1=3,a_{n+1}=a_n! \]\((a_n,a_{n+1})\) 之间必有素数,且区间之间无交集,将自然数集分成无穷多个区间,从而有无穷多个素数

习题1.2.1

\(n\) 是正整数, 证明: \(\dfrac{21 n+4}{14 n+3}\) 是既约分数.


证明:反证法,假设 \(\dfrac{21n+4}{14n+3}\) 不是既约分数,则 \(21n+4\)\(14n+3\) 有共同素因子 \(d\) ,即 \[ \because (21n+4,14n+3)=d,21n+4=dm,14n+3=dn\\ \therefore 3\cdot dn-2\cdot dm=42n+9-(42n+8)=1=d(3n-2m) \]\(d\) 为大于 \(1\) 的素因子,\(1\nmid d\),从而与假设矛盾,则 \(\dfrac{21n+4}{14n+3}\) 是既约分数

习题1.2.2

\(m, n\) 为正整数, \(m\) 为奇数,证明: \[ \left(2^{m}-1,2^{n}+1\right)=1 \]


证明:令 \((2^{m}-1,2^{n}+1)=d\),且有 \(2^{m}-1=dx,2^{n}+1=dy,x,y\in Z\)\(d\) 为奇数 \[ \because 2^m=dx+1,2^n=dy-1\ \therefore 2^{mn}=(dx+1)^{n}=(dy-1)^m \] 将其二项式展开有 \(\displaystyle \sum_{i=0}^nC_n^i(dx)^i=\sum_{i=0}^mC_m^i(dy)^i(-1)^{m-i}\),两边同时除以 \(d\) 得到 \[ \displaystyle \sum_{i=1}^nC_n^id^{i-1}x^{i}+\dfrac{1}{d}=\sum_{i=1}^mC_m^id^{i-1}y^i(-1)^{m-i}+(-1)^m\cdot d^{-1} \] 由于 \(m\) 是奇数,若 \(d\geq 3\),两边取整有 \(\dfrac{1}{d}=1-\dfrac{1}{d}\),矛盾,从而 \(d=1\)

习题1.2.3

\(m, n, a\) 均为正整数, \(a \geqslant 2\), 证明: \[ \left(a^{m}-1, a^{n}-1\right)=a^{(m, n)}-1 \text {. } \]


证明:令 \((m,n)=d\),且有 \(m=dx,n=dy,(x,y)=1\),由引理 \(1.1.1\)\[ a^{d}-1\mid a^{dx}-1,\ a^{dx}-1=(a^d-1)\cdot(1+a^d+a^{2d}+\cdots+a^{(x-1)d})\\ a^{d}-1\mid a^{dy}-1,\ a^{dy}-1=(a^d-1)\cdot(1+a^d+a^{2d}+\cdots+a^{(y-1)d}),\\ \] 考虑辗转相除法,不妨设 \(x>y\)\[ 1+a^d+a^{2d}+\cdots+a^{(x-1)d}=a^{(x-y)d}(1+a^d+\cdots+a^{(y-1)d})+1+a^d+a^{2d}+\cdots+a^{(x-y-1)d} \] 继续判断 \(x\)\(y\) 的大小关系,辗转相除,而 \((x,y)=1\),则 \[ (1+a^d+a^{2d}+\cdots+a^{(x-1)d},1+a^d+a^{2d}+\cdots+a^{(y-1)d})=1 \] 从而 \(a^{d}-1=a^{(m,n)}-1\) 即为 \(a^{m}-1\)\(a^{n}-1\) 的最大公因数

\(\large\textcolor{blue}{初等数论第二次作业}\ \ \ \ \ \ _\textcolor{blue}{2022.3.2}\)

习题1.2.4

\(a,b,c\in Z,a\neq 0\)。则 \(a\mid bc\) 当且仅当 \(\dfrac{a}{(a,b)}\mid c\)


证明:取 \(d=(a,b)\),令 \(a=dx,b=dy\),且有 \((x,y)=1\)

\((1)\)\(a\mid bc\)\(\exists \ t\in Z,\ bc=at\ \therefore yc=xt \ \ x\mid yc\),由于 \((x,y)=1\),得 \(x\mid c\),即 \(\dfrac{a}{(a,b)}\mid c\)

\((2)\)\(\dfrac{a}{(a,b)}\mid c\),则 \(\exists \ t\in Z,\ c=\dfrac{a}{(a,b)}\cdot t\ \therefore (a,b)\cdot c=at=dc=dxt\),即 \(c=xt\), 两边乘

\(b=dy\)\(bc=xydt=ayt\),从而 \(a\mid bc\)

综上,\(a\mid bc \Longleftrightarrow \dfrac{a}{(a,b)}\mid c\)

习题1.2.5

\(m\)\(n\) 是互素的正整数,证明:

\(\ \ \ (1)\) 对每个整数 \(a\)\((a,mn)=(a,m)(a,n)\)

\(\ \ \ (2)\) \(mn\) 的每个正因子 \(d\) 均可唯一地表示成 \(d=d_1d_2\),其中 \(d_1\)\(d_2\) 分别为 \(m\)\(n\) 的正因子。


证明:\((1)\)\((a,mn)=(a,m)\cdot(\dfrac{a}{(a,m)},\dfrac{mn}{(a,m)})\),由于 \(\dfrac{a}{(a,m)}\)\(\dfrac{m}{(a,m)}\) 互素,则 \[ (a,mn)=(a,m)\cdot (\dfrac{a}{(a,m)},\dfrac{mn}{(a,m)})=(a,m)\cdot \normalsize (\dfrac{a}{(a,m)},n)令\ a=k_1x,m=k_2x\ \\ \because (m,n)=1\therefore(k_2x,n)=(x,n)=1,(\dfrac{a}{(a,m)},n)=(k_1,n)=(k_1x,n)=(a,n) \] 则有 \((a,mn)=(a,m)(a,n)\)

\((2)\) 由算术基本定理,令 $m=p_1^{_1}p_2^{_2}p_r^{_r},n=q_1^{_1}q_2^{_2}q_k^{_k},1_i,_i $,由 \((m,n)=1\)

\(\forall \ 1\leq i\leq r,1\leq j\leq k,p_i\neq q_j\),否则设相等素数为 \(p_{i_{0}}\) \((m,n)\geq p_{i_0}>1\) 矛盾,则有新序列集合

\(\mathbb{S}=\{p_1,\cdots,p_r,q_1,\cdots,q_k\}\),有 \(mn=p_1^{\alpha_1}\cdots p_r^{\alpha_r}\cdot q_1^{\beta_1}\cdots q_k^{\beta_k}\),若 \(d\)\(mn\) 的因子,则

总有 \(d=p_1^{\gamma_1}\cdots p_{r}^{\gamma_{r}}\cdot q_{1}^{\lambda_{1}}\cdots q_{k}^{\lambda_{k}},0\leq\gamma_{i},\lambda_{i}\),则分离 \(d_1=p_1^{\gamma_1}\cdots p_{r}^{\gamma_{r}},d_2=q_{1}^{\lambda_{1}}\cdots q_{k}^{\lambda_{k}}\)

使得 \(d=d_1d_2\),且由于 \(p_i\)\(q_i\) 完全不同,该分解存在且唯一

习题1.2.6

\(n\) 为正整数,\(a,b\) 是不全为零的整数,证明:

\(\ \ \ (1)\) \((a^n,b^n)=(a,b)^n\)

\(\ \ \ (2)\)\(a\)\(b\) 是互素的正整数,\(ab=c^n,c\in Z\),则 \(a\)\(b\) 都是正整数的 \(n\) 次方幂。


证明:\((1)\)\((a,b)=d\),则有 \(a=dx,b=dy,x,y\in Z\),从而 \(a^n=d^nx^n,b^n=d^ny^n\)

从而 \(d^n\mid a^n,d^n\mid b^n,d^n\mid (a^n,b^n)\),则令 \((a^n,b^n)=td^n,t\in Z\),代入上式有

\(\dfrac{a^n}{(a^n,b^n)}=tx^n,\ \dfrac{b^n}{(a^n,b^n)}=ty^n\),结合 \((x^n,y^n)=1\)\(1=(\dfrac{a^n}{(a^n,b^n)},\dfrac{b^n}{(a^n,b^n)})=t\)

\(\therefore (a^n,b^n)=1\cdot d^n=(a,b)^n\)

\((2)\) 由算术基本定理,令 $a=p_1^{_1}p_2^{_2}p_r^{_r},b=q_1^{_1}q_2^{_2}q_k^{_k},1_i,_i $,由 \((a,b)=1\)

\(\forall \ 1\leq i\leq r,1\leq j\leq k,p_i\neq q_j\)​,否则设相等素数为 \(p_{i_{0}}\) \((m,n)\geq p_{i_0}>1\) 矛盾,则有新序列集合

\(\mathbb{S}=\{p_1,\cdots,p_r,q_1,\cdots,q_k\}\),有 \(ab=p_1^{\alpha_1}\cdots p_r^{\alpha_r}\cdot q_1^{\beta_1}\cdots q_k^{\beta_k}=c^n\),同理对 \(c\) 分解有

\(c=m_1^{\gamma_1}\cdots m_l^{\gamma_{l}}\),代入有 \(p_1^{\alpha_1}\cdots p_r^{\alpha_r}\cdot q_1^{\beta_1}\cdots q_k^{\beta_k}=m_1^{n\gamma_1}\cdots m_l^{n\gamma_{l}}\),由算术基本定理有

\(\alpha_{i}=n\gamma_{a_{i}},\beta_{i}=n\gamma_{b_{i}}\),则 \(a\)\(b\) 均能写成某个正整数的 \(n\) 次幂

习题1.29

\(963x+657y=(963,657)\) 的全部整数解。


解:由欧几里得辗转相除法有 \[ 963=657+306\longrightarrow 657=306\times 2+45\longrightarrow 306=45\times 6+36\\ \longrightarrow 45=36\times 1+9\longrightarrow 36=9\times 4 \] 从而 \((963,657)=9\),依次反向欧几里得算法有 \[ 9=45-36\times 1=45-(306-45\times 6)=45\times 7-306=(657-306\times 2)\times7-306\\ =657\times 7-306\times 15=657\times 7-306\times 15=657\times 7-(963-657)\times 15\\ =963\times(-15)+657\times22 \]\(963x+657y=(963,657)=9\),可以化为 \(107x+73y=1\),从而其解为 \[ \begin{cases} x=-15+73t\\ y=22-107t \end{cases} \ \ \ \ t\in Z \]

习题1.2.10

求下列方程的全部整数解:

\(\ \ \ (1)\) \(6x+20y-15z=23\)

\(\ \ \ (2)\) \(25x+13y+7z=2\)


\((1)\)\((20,15)=5\),则 \(6x+20y-15z=23\Longleftrightarrow \begin{cases}20y-15z=5\omega\cdots\boldsymbol ①\\6x+5\omega=23\cdots\boldsymbol ②\end{cases}\)

\(\boldsymbol ②\),由 \(6=5\times 1+1\),则 \(1=6\times1+5\times(-1)\),则 \(\boldsymbol ②\) 解为 \(\begin{cases}x=23+5t\\\omega=-23-6t\end{cases}\)

代入 \(\boldsymbol ①\) 变形为 \(4y-3z=\omega=-23-6t\),由 \(4=3\times1+1\),则解 \(\begin{cases}y=\omega+3u\\z=\omega+4u\end{cases}\)

从而 \(6x+20y-15z=23\) 的通解为 \(\begin{cases}x=23+5t\\y=-23-6t+3u\\z=-23-6t+4u\end{cases}\ \ \ \ \ t,u\in Z\)

\((2)\)\((13,7)=1\),则 \(25x+13y+7z=2\Longleftrightarrow \begin{cases}13y+7z=\omega\cdots\boldsymbol ①\\25x+\omega=1\cdots\boldsymbol ②\end{cases}\)

\(\boldsymbol ②\),由 \(25=25\times 1\),则 \(1=25\times 0+1\times1\),则 \(\boldsymbol ②\) 解为 \(\begin{cases}x=t\\\omega=1-25t\end{cases}\)

代入 \(\boldsymbol ①\) 变形为 \(13y+7z=\omega=1-25t\),由 \(1=7\times 2-13\),则解 \(\begin{cases}y=-\omega+7u\\z=2\omega-13u\end{cases}\)

从而 \(25x+13y+7z=2\) 的通解为 \(\begin{cases}x=t\\y=-1+25t+7u\\z=2-50t-13u\end{cases}\ \ \ \ \ t,u\in Z\)

习题1.2.12

\(f(x)=x^n+a_1x^{n-1}+\cdots +a_{n-1}x+a_n\) 为首项系数为 \(1\) 的整系数多项式,

\(f(x)\) 的每个有理根必为整数。


证明:若整系数多项式 \(x^n+a_1x^{n-1}+\cdots+a_{n-1}x+a_n=0\) 有理根为 \(\dfrac{p}{q}(p,q\in Z,q\neq 0)\) \[ \because (p,q)=1,(\dfrac{p}{q})^{n}+a_1(\dfrac{p}{q})^{n-1}+\cdots+a_{n-1}\dfrac{p}{q}+a_n=0\\ \therefore p^{n}=-(a_1p^{n-1}q+\cdots+a_{n-1}pq^{n-1}+a_nq^{n})\therefore q\mid p^{n}\Longrightarrow q=1 \] 从而所有有理根必为整数

习题1.2.13

\(m\)\(n\) 为正整数,则在 \(n,2n,\cdots,mn\)\(m\) 个数当中恰有 \((m,n)\) 个是 \(m\) 的倍数。


证明:令 \(d=(m,n)\),且 \(m=dx,n=dy,(x,y)=1\),从而 \(kn,k=1,2,\cdots,m\) 满足

\(m\mid kn,dx\mid kdy\Longrightarrow x\mid ky\),由于 \((x,y)=1\),则 \(x\mid k\),即 \(k\)\(x\) 的倍数,即 \(k=xt,t\in Z\)

\(m=dx\),恰好为 \(1\cdot x,2\cdot x\cdots ,d\cdot x\) ,恰共有 \(d =(m,n)\)

习题1.2.16

\(m\)\(n\) 是互素的非零整数。证明对整数 \(a\),若 \(m\mid a,n\mid a\),则 \(mn\mid a\)


证明:已知 \((m,n)=1\),由 \(m,n\mid a\),设 \(a=mx=ny,x,y\in Z\),有 \((mx,nx)=x(m,n)=x\)

代入 \(mx=ny\)\(x=(ny,nx)=n(y,x)\),则 \(n\mid x,mn\mid mx=a\),即 \(mn\mid a\)

习题1.3.3

\(a,b,c\) 为正整数,证明

\(\ \ \ (1)\) \((a,[b,c])=[(a,b),(a,c)]\)

\(\ \ \ (2)\) \([a,(b,c)]=([a,b],[a,c])\)


证明:由算术基本定理,设 \(a,b,c\) 三者分解下所有质因数的交集为 \(\{p_1\cdots p_N\}\),且三者对应的

幂次为 \(\{a_n\},\{b_n\},\{c_{n}\},0\leq a_i,b_i,c_i\in Z\),即 \(a,b,c=\displaystyle \prod p_{i}^{a_i,b_i,c_i}\)

\((1)\) \((a,[b,c])=\displaystyle \prod p_{i}^{\min(a_i,\max({b_i,c_i}))}\)\([(a,b),(a,c)]=\displaystyle \prod p_{i}^{\max(\min(a_i,b_i),\min(a_i,c_i))}\)

\((a,[b,c])=[(a,b),(a,c)]\Longleftrightarrow \min(a_i,\max(b_i,c_i))=\max(\min(a_i,b_i),\min(a_i,c_i))\)

而对后者,调换 \(b\)\(c\) 对等式左右两边都不变,不妨设 \(b\leq c\),从而分类以下三种情况: \[ \begin{gathered} i) a \leq b \leq c;\ \ \ \ ii) b \leq a \leq c;\ \ \ \ iii) b \leq c \leq a\\ i) \max \{a, \min \{b, c\}\}=\max \{a, b\}=b, \min \{\max \{a, b\}, \max \{a, c\}\}=\min \{b, c\}=b;\\ ii) \max \{a, \min \{b, c\}\}=\max \{a, b\}=a, \min \{\max \{a, b\}, \max \{a, c\}\}=\min \{a, c\}=a;\\ iii) \max \{a, \min \{b, c\}\}=\max \{a, b\}=a, \min \{\max \{a, b\}, \max \{a, c\}\}=\min \{a, a\}=a; \end{gathered} \] 从而后者成立,有 \((a,[b,c])=[(a,b),(a,c)]\) 成立

\((2)\) 仿照 \(\small (1)\) 讨论 \(\max(a_i,\min(b_i,c_i))=\min(\max(a_i,b_i),\max(a_i,c_i))\),也不妨设 \(b\leq c\) \[ \begin{gathered} i) a \leq b \leq c;\ \ \ \ ii) b \leq a \leq c;\ \ \ \ iii) b \leq c \leq a\\ i) \min \{a, \max \{b, c\}\}=\min \{a, c\}=a, \max \{\min \{a, b\}, \min \{a, c\}\}=\max \{a, a\}=a;\\ ii) \min \{a, \max \{b, c\}\}=\min \{a, c\}=a, \max \{\min \{a, b\}, \min \{a, c\}\}=\max \{b, a\}=a;\\ iii) \min \{a, \max \{b, c\}\}=\min \{a, c\}=c, \max \{\min \{a, b\}, \min \{a, c\}\}=\max \{b, c\}=c;\\ \end{gathered} \] 从而同理可得 \([a,(b,c)]=([a,b],[a,c])\)

习题1.3.4

不存在整数 \(m\geq 2\) ,使得 \(m^2\mid n\) 的正整数 \(n\) 为无平方因子,证明:

\(\ \ \ (1)\) 正整数 \(n\) 是无平方因子当且仅当 \(n=1\) 或者是不素因子的乘积;

\(\ \ \ (2)\) 每个正整数 \(n\) 可以唯一地表示成 \(n=m^2\cdot n'\),其中 \(m^2\) 为平方数,而 \(n'\) 为无平方因子整数。


证明:\((1)\)\(n=1\)\(\forall m\geq 2,\ m^2\nmid n\),从而 \(1\) 为无平方因子,若 \(n\geq 2\),由算术基本定理

\(n=p_1^{\alpha_1}\cdot p_2^{\alpha_2}\cdots p_r^{\alpha_r}\),若 \(\exists\ \alpha _i\geq 2\),则取 \(p_i^{2}\mid p_{i}^{\alpha_{i}}\mid n\),则不是无平方因子,从而

\(\forall \ \alpha_{i}\leq 1\),而 \(\alpha_i=0\) 无意义(\(p_{i}^{0}=1\)),从而此时 \(n\) 为不同素因子乘积

又若 \(n=1\) 或者 \(n\) 为不同素因子乘积,显然满足无平方因子条件,从而两者等价

\((2)\)\(n\) 为无平方因子,则 \(n=1^2\cdot n'\) 满足条件,若 \(n\) 不为无平方因子,由 \(\small (1)\) 得,\(\exists\ \alpha_i\geq 2\)

则挑选 \(\mathbb{A}=\{i\mid \alpha_i\geq 2\}\),对 \(\forall\ x\in \mathbb{A}\) 取 ${i}=2,{i}=_i-_i=0,1 $

则所有非 \(\mathbb{A}\) 中的 \(\alpha_{j}\) 以及 \(\gamma_{i}\) 对应素数幂次的乘积组成无平方因子 \(n'=\displaystyle \prod_{\alpha_{j}\notin\mathbb{A}}p_{j}^{\alpha_{j}}\cdot \prod_{\alpha_{i}\in\mathbb{A}}p_{i}^{\gamma_{i}}\)

其余组成完全平方数 \(m^2=\displaystyle \prod_{\alpha_{i}\in\mathbb{A}}p_{i}^{\beta_{i}}=\prod_{\alpha_{i}\in\mathbb{A}}p_{i}^{2\lfloor\frac{\alpha_i}{2}\rfloor}\),即 \(m=\displaystyle \prod_{\alpha_{i}\in\mathbb{A}}p_{i}^{\lfloor\frac{\alpha_i}{2}\rfloor}\),使得 \(n=m^2\cdot n'\)

下证其唯一性:若有两组 \((m_1,n'_1),(m_2,n_2')\) 使得 \(n=m_1^2n_1'=m_2^2n_2'\),由 \(\small(1)\)

\(n_1\)\(n_2\)\(1\) 或者是不同素数的乘积,若两者不相同,则两边消去相同部分后总会剩余成单素数

\(n''=m_1^2\displaystyle \prod p_i=m_2^2\prod q_i\),考察 \(p_i\) 对应的幂次有左侧为奇次幂,右侧为偶次幂

这与 \(n''\) 的算术基本定理矛盾,从而每个正整数能唯一地表示成完全平方数和无平方因子的乘积

\(\large\textcolor{blue}{初等数论第三次作业}\ \ \ \ \ \ _\textcolor{blue}{2022.3.8}\)

习题1.2.1

\(m\) 为正整数, \((a, m)=1\). 我们用 \(a^{-1}\) 表示同余方程 \(a x \equiv 1\) \((\bmod m)\) 的任何一个整数解 (即

\(\left.a^{-1} \in \mathbf{Z}, a a^{-1} \equiv 1(\bmod m)\right)\). 证明:

  1. \((a, m)=(b, m)=1\), 则 \(a \equiv b(\bmod m)\) $$ \(a^{-1} \equiv\) \(b^{-1}(\bmod m)\)

  2. \(\{r_{1}, r_{2}, \cdots, r_{\varphi(m)}\}\) 是模 \(m\) 的缩系, 则 \(\{r_{1}^{-1}, r_{2}^{-1}, \cdots, r_{\varphi(m)}^{-1}\}\) 也是模 \(m\) 的缩系。


证明:\((1)\)\(a\equiv b(\mbox{mod}\ m)\),即 \(a-b=k_1m,k_1\in Z\),令 \(a^{-1}\)\(b^{-1}\) 中最小的正整数为

\(a_{-},b_{-}\),对上式乘以 \(a_{-},b_{-}\)\(aa_{-}b_{-}-a_{-}(bb_{-})=k_1ma_{-}b_{-}\),两边对 \(m\) 求同余有

\(a_{-}\equiv b_{-}(\mbox{mod}\ m)\),而 \(\overline{a^{-1}}=a_{-}+km,k\in Z,\overline{b^{-1}}=b_{-}+km,k\in Z\)

\(a\equiv b(\mbox{mod}\ m)\Longrightarrow a^{-1}\equiv b^{-1}(\mbox{mod}\ m)\),由对称性(\((a^{-1})^{-1}=a\)),两者等价

\((2)\)\(r_{i}r_{i}^{-1}\equiv 1(\mbox{mod}\ m)\),若 \((r_{i}^{-1},m)\geq 2\),则 \((r_{i}r_{i}^{-1},m)\geq 2\),与 \(r_{i}r_{i}^{-1}\equiv 1(\mbox{mod}\ m)\)

矛盾,从而所有的 \(r_{i}^{-1}\)\(m\) 互素;又若 $ ij,r_{i}{-1}=r_{j}{-1} $,由 \((1)\)\(r_{i}=r_{j}\) ,矛盾

\(m\) 的缩系有 \(\varphi(m)\) 个,从而 \(\{r_{1}^{-1},\cdots ,r_{\varphi (m)}^{-1}\}\) 也为缩系

习题1.2.2

设正整数 \(n\) 的十进制表示为

\[ n=a_{0}+a_{1} \cdot 10+a_{2} \cdot 10^{2}+\cdots+a_{k} \cdot 10^{k}, \]

证明: \(\left\{\begin{array}{l}n \equiv a_{0}+a_{1}+\cdots+a_{k}(\bmod 9), \\ n \equiv a_{0}-a_{1}+a_{2}-\cdots+(-1)^{k} a_{k}(\bmod 11) .\end{array}\right.\) 用这些结果来计算 \(12345 \times 6789\)\(9\)

\(11\) 除所得的余数。


证明:由 \(u=\displaystyle \sum_{i=0}^ka_i10^{i}\),且 \(10^{i}\equiv(9+1)^{i}\equiv1+9^{i}\equiv1(\mbox{mod}\ m)\)

\(10^{i}\equiv (11-1)^{i}\equiv(-1)^{i}+11^{i}\equiv (-1)^{i}(\mbox{mod}\ m)\),代入则得到

\[ \left\{\begin{array}{l}n \equiv \displaystyle \sum_{i=0}^ka_i(\bmod 9), \\ n \equiv \displaystyle \sum_{i=0}^k(-1)^{i}a_i(\bmod 11) .\end{array}\right. \] 代入计算 \(12345\times 6789\equiv 15\times 30\equiv 0(\mbox{mod}\ 9)\)\(12345\times 6789\equiv 3\times2\equiv 6(\mbox{mod}\ 11)\)

习题1.2.6

证明:当 \(n \geqslant 3\) 时, \(\varphi(n)\) 为偶数。


证明:\(\boldsymbol ①\)\(n\) 有奇素数因子,则 \(\varphi(n)=\displaystyle \prod_{i}(p_{i}^{\alpha_{i}}-p_{i}^{\alpha _{i}-1})\) 中对奇素数因子 \((p_{i}^{\alpha_{i}}-p_{i}^{\alpha _{i}-1})\)

总为偶数,从而此时 $(n)( 2) $

\(\boldsymbol ②\)\(n\) 没有奇素数因子,由于 \(n\geq 3\),则 \(n=2^{k},k\geq 2\),此时 \(\varphi(n)=2^{k-1}\) 也为偶数

从而 \(\forall\ n\geq 3\)\(\varphi(n)\) 为偶数

习题1.2.7

\(m\)\(n\) 是正整数,\(m=n t(t \in \mathbf{Z})\). 证明: 模 \(n\) 的每个同余类都是模 \(m\)\(t\) 个同余类之并。


证明:对 \(m\) 中特定 \(\overline{a}\) ,取 \(\overline{a},\overline{a}+\overline{n},\cdots ,\overline{a}+(t-1)\overline{n}\)\(t\) 个同余类,这些元素均模 \(n\) 余数为 \(a\)

\(n\),而 \(\overline{a}\) 取遍 \(0,1,\cdots,n-1\),得到模 \(n\) 的每个同余类。

习题1.2.10

\(a, m \in \mathbf{Z}, m \geqslant 2,(a, m)=1\). 计算 \(\displaystyle \sum_{x=0}^{m-1}\left[\frac{a x}{m}\right]\).


解:由于 \(\{ax\big |x=0,1,\cdots ,m-1\}\)\(m\) 的完系,则拆开 \(\displaystyle \left[\frac{a x}{m}\right]=\dfrac{ax}{m}-\{\dfrac{ax}{m}\}\),则 \[ \begin{gathered} \displaystyle \sum_{x=0}^{m-1}\left[\frac{a x}{m}\right]=\dfrac{a}{m}\sum_{x=0}^{m-1}x-\dfrac{1}{m}(0+1+\cdots +(m-1))=\dfrac{a}{m}\cdot\dfrac{m(m-1)}{2}-\dfrac{m(m-1)}{2m}\\=\dfrac{a(m-1)}{2}-\dfrac{m-1}{2}=\dfrac{(a-1)(m-1)}{2} \end{gathered} \]

更一般地,对 \(\forall \ a,m\in \mathbf{Z}\),都有 \(\displaystyle \sum_{x=0}^{m-1}\left[\frac{a x}{m}\right]=\dfrac{am-a-m+(a,m)}{2}\)

习题2.2.1

\(\alpha\) 是环 \(Z_{m}\) 中非零元素。若存在 \(Z_{m}\) 中非零元素 \(\beta(\neq \overline{0})\), 使得 \(\alpha \beta=\overline{0}\), 称 \(\alpha\) 是零因子,证明:

\((1)\) 非零元素 \(\alpha\) 是零因子当且仅当 \(\alpha\) 不可逆. 从而 \(Z_{m}\) 由彼此不同的三类元素构成:\(\overline{0}, \varphi(m)\) 个可逆

元和 \(m-\varphi(m)-1\) 个零因子;

\((2)\) \(Z_{m}\) 中没有零因子当且仅当 \(m\) 是素数。


证明:\((1)\)\(\alpha (\neq \overline{0})\) 是零因子,反证法,若 $$ 可逆,则 \(\alpha\alpha^{-1}\equiv 1(\mbox{mod}\ m)\),两边乘以 \(\beta(\neq \overline{0})\)

\(\alpha \beta \alpha^{-1}\equiv \beta(\mbox{mod}\ m)\),左边为 \(0\),右边不为 \(0\),矛盾,从而 $$ 不可逆;又若 $$ 不可逆,则有

\(\forall \ \beta\neq 0,\alpha \beta\neq 1\)。则考虑 \(\{0,\overline{\alpha },\overline{2\alpha},\cdots,\overline{(m-1)\alpha}\}\) ,该集合不为模 \(m\) 的同余类,否则一定能

找到 \(\overline{1}\) 与 $$ 不可逆的条件矛盾,则 \(\exists\ i,j\in[1,m-1],\overline{i\alpha}=\overline{j\alpha}\),从而取 \(\beta=i-j\),$$ 为零因子

\((2)\)\(m\) 为素数,\(\varphi(m)=m-1\),则有 \(m-\varphi(m)-1=0\) 个零因子

\(m-\varphi(m)-1=0\) 成立,则 \(\varphi(m)=m-1\),若 \(m\) 为合数,\(m=ab\),则 \(a\)\(b\) 为零因子,

\(\varphi(m)\leq m-1-2\) 矛盾

习题2.2.2

\((1)\) 对于环 \(Z_{m}\) 中任何元素 \(\alpha\)\(m\)\(\alpha\) 相加为 \(\overline{0}\).

\((2)\)\(p\) 为素数. 对于域 \(Z_{p}\) 中非零元素 \(\alpha\) 和正整数 \(n\),证明: \(n\)\(\alpha\) 相加为 \(\overline{0}\) 当且仅当 \(p \mid n\)


证明:\((1)\)\(\alpha_{i}=\alpha_{0}+k_{i}\cdot m\),这些 \(m\) 个 $$ 之和为 \(m\alpha_{0}+m\displaystyle \sum_{i=1}^{m}k_{i}\),均模 \(m\)\(0\)

而且对于 \(\overline{0}\) 中的所有元素 \(Km\),都能找到 \(k_{i}\) 使得 \(\alpha_{0}+\displaystyle \sum_{k=1}^{m}k_{i}=K\),则相加为 \(\overline{0}\)

\((2)\)\(p\mid n\),有 \(n\) 个 $$ 相加后模 \(m\)\(0\),则 \(n\)\(\alpha\) 相加为 \(\overline{0}\);又若 \(n\)\(\alpha\) 相加为 \(\overline{0}\)

\(\alpha_{i}=\alpha_{0}+k_{i}\cdot m\),相加后模 \(m\)\(n\alpha_{0}\equiv 0(\mbox{mod}\ p)\),而 \(\alpha_{0}=0,1,\cdots ,p-1\)

\((\alpha_{0},p)=1\),则 \(n\equiv 0(\mbox{mod}\ p)\),即 \(p\mid n\)

习题2.2.3

证明当 \(p\) 为奇素数时,\(2^{p-1} \cdot\left(\left(\dfrac{p-1}{2}\right) !\right)^{2} \equiv(-1)^{\frac{p+1}{2}}(\bmod p)\)


证明:\(2^{p-1}(\dfrac{p-1}{2}\cdot\dfrac{p-3}{2}\cdots\dfrac{4}{2}\cdot \dfrac{2}{2})^{2}=(p-1)^{2}(p-3)^2\cdots 4^2\cdot 2^2\),使用补偿法\[ \begin{gathered} (p-1)^2(p-3)^2\cdots4^2\cdot2^2\equiv2\cdot 4\cdot 6\cdots(p-1)\cdot (p-1)\cdot (p-3)\cdots 4\cdot 2\\ \equiv2\cdot 4\cdot 6\cdots (p-1)\cdot (p-(p-1))\cdot (p-(p-3))\cdots (p-4)\cdot (p-2)\cdot (-1)^{\frac{p-1}{2}}\\ \equiv2\cdot 4\cdots (p-1)\cdot 3\cdot 5\cdots (p-2)\cdot (-1)^{\frac{p-1}{2}} \equiv (p-1)!(-1)^{\frac{p-1}{2}}\equiv (-1)^{\frac{p+1}{2}}(\mbox{mod}\ p) \end{gathered} \] 其中使用威尔逊定理,对素数 \(p\) ,有 \((p-1)!\equiv-1(\mbox{mod}\ p)\)

习题2.2.4

对于整数 \(m \geqslant 2\), 证明: \((m-1) ! \equiv-1(\bmod m)\) 当且仅当 \(m\) 是素数。 (判别 \(m\) 是否为素数)


证明:若 \(m\) 为素数,则由威尔逊定理得 \((m-1)!\equiv -1(\mbox{mod}\ m)\);若 \(m\) 为合数,则 \(m=ab\)

其中 \(2\leq a\leq b\),则 \(1\leq a,b\leq m-1\),从而 \((m-1)!\) 中一定能凑齐 \(m\),此时模为 \(0\)

习题2.2.5

\(Z_{m}^{*}=\left\{\alpha_{1}, \cdots, \alpha_{\phi(m)}\right\}\),则 \(Z_{m}^{*}=\left\{\alpha_{1}^{-1}, \cdots, \alpha_{\phi(m)}^{-1}\right\}\)。如何将它转述成同余的语言?


证明:缩系等价性证明在习题 \(1.2.1\) 中。定义 \(\alpha_{i}x_i\equiv 1(\mbox{mod}\ m)\)\(x_{i}\) 的可取值为 \(\alpha_{i}^{-1}\)

转述为 \((\alpha_{i},m)=1\),所有 \(\alpha_{i}\) 构成 \(Z_{m}^*\),由 \(\alpha_{i}x_{i}\equiv 1(\mbox{mod}\ m)\)中,所有 \(x_{i}\) 也构成 \(Z_{m}^*\)

习题2.3.1

\(n\)\(m\) 是互素的正整数,证明: \(m^{\varphi(n)}+n^{\varphi(m)}\equiv1(\bmod m n)\)


证明:由欧拉-费马定理,\(m^{\varphi(n)}\equiv1(\mbox{mod}\ n),n^{\varphi(m)}\equiv1(\mbox{mod}\ m)\),且由\(\varphi(x)\geq 1\),得

\(m^{\varphi(n)}\equiv0(\mbox{mod}\ m),n^{\varphi(m)}\equiv0(\mbox{mod}\ n)\),从而 \[ m^{\varphi(n)}+n^{\varphi(m)}\equiv 1(\bmod m),m^{\varphi(n)}+n^{\varphi(m)}\equiv 1(\bmod n) \]\(m^{\varphi(n)}+n^{\varphi(m)}=1+mk_1=1+nk_2\),由于 \((m,n)=1\),则 \(n\mid k_1\),原式成立

习题2.3.2

\((1)\) 对每个与 \(10\) 互素的整数 \(a\), 证明: \(a^{20} \equiv 1(\bmod 100)\)

\((2)\)\(3^{193}\) 的十进制表达式中的个位和十位数字。


解:\((1)\)\(\varphi(4)=2,\varphi(25)=20\),结合 \((a,4)=(a,25)=1\)\(a^{20}\equiv 1(\bmod 25)\)

\(a^2\equiv 1(\bmod 4)\)\(a^{20}\equiv (a^2)^{10}\equiv 1(\bmod 4)\),而 \((4,25)=1\),则 \(a^{20}\equiv 1(\bmod 100)\)

\((2)\)\(3^{193}\equiv 3^{160}\cdot 3^{33}\equiv 27^{11}\equiv 2^{11}\equiv 23(\bmod 25)\),且 \(3^{193}\equiv (-1)^{193}\equiv 3(\bmod 4)\)

从而最后两位为 \(23\)

习题2.3.3

\((1)\)\(a, b \in \mathbf{Z}, n \geqslant 1, p\) 为素数。如果 \(a \equiv b\left(\bmod p^{n}\right)\),证明:对每个整数 \(k \geqslant 0\)

\[ a^{p^{k}} \equiv b^{p^{k}}\left(\bmod p^{n+k}\right) \] \((2)\) 证明: 对每个奇数 \(a\)\(k \geqslant 1, a^{2^{k}} \equiv 1\left(\bmod 2^{k+2}\right)\).


证明:\((1)\)\(b=a+x\cdot p^{n},x\in Z\),代入使用二项式展开有 \[ b^{p^{k}}-a^{p^{k}}=(a+x\cdot p^{n})^{p^{k}}-a^{p^{k}}=\displaystyle \sum_{i=1}^{k}C_{p^{k}}^{i}b^{i}(x\cdot p^{n})^{p^{k}-i} \] 其中 \(p^{k}\mid C_{p^{k}}^{i}\),且有 \(p^{n}\mid (x\cdot p^{n})^{p^{k}-i}\),从而 \(a^{p^{k}} \equiv b^{p^{k}}\left(\bmod p^{n+k}\right)\)

\((2)\)\(a\) 为奇数,令 \(a=2i+1\),则 \(a^{2}=4i(i+1)+1\),且 \(i\)\(i+1\) 一奇一偶,从而

\(a^{2}\equiv 1(\bmod 2^{3})\),代入 \(\small (1)\) 中结论有 \((a^2)^{2^k}\equiv 1(\bmod 2^{3+k})\),令 \(k'=k+1\geq 1\) 则有

\(k' \geqslant 1, a^{2^{k'}} \equiv 1\left(\bmod 2^{k'+2}\right)\),得证

\(\large\textcolor{blue}{初等数论第四次作业}\ \ \ \ \ \ _\textcolor{blue}{2022.3.15}\)

习题2.1.3

解下列同余方程。

\((1)\) \(8 x \equiv 5(\bmod 23)\); \((2)\) \(60 x \equiv 7(\bmod 37)\).


解:\((1)\)\((8,23)=1\),则在 \(Z_{23}\) 中有唯一解,利用费马小定理 \[ x\equiv 8^{\varphi(23)-1}\cdot 5\equiv 8^{21}\cdot 5\equiv 64^{10}\cdot 40\equiv 5^{10}\cdot( -6)\equiv -(25)^{5}\cdot 6\equiv -2^{5}\cdot 6\equiv 15\pmod {23} \] \((2)\)\((60,37)=1\),则在 \(Z_{37}\) 中有唯一解,利用费马小定理 \[ \begin{gathered} x\equiv 60^{\varphi(37)-1}\cdot 7\equiv(-14)^{35}\cdot 7\equiv 196^{17}\cdot 13\equiv 11^{17}\cdot 13\\\equiv 10^{8}\cdot (-5)\equiv -5\cdot 11^4 \equiv -5\cdot 10^2\equiv (-5)\cdot (-11)\equiv 18\pmod{37} \end{gathered} \]

习题2.1.4

对每个整数 \(n\),证明: \[ \begin{gathered} (1) n^{2} \neq 2(\bmod 3);&(2) n^{2} \equiv 0 或 1(\bmod 4);\\ (3) n^{3} \equiv 0,1 或 -1(\bmod 9) ;&(4) n^{4} \equiv 0 或 1(\bmod 16). \end{gathered} \]


证明:\((1)\) 由费马小定理,若 \((n,3)=1,n^{\varphi(3)}\equiv n^2\equiv 1\pmod 3\),且若 \((n,3)>1\),则 \(3\mid n\)

此时有 \(n^2\equiv 0\pmod 3\),则 \(n^2\neq2 \pmod 3\)

\((2)\) 由费马小定理,若 \((n,4)=1,n^{\varphi(4)}\equiv n^2\equiv 1\pmod 4\),且若 \((n,4)>0\),有 \(2\mid 4\)

此时有 \(n^2\equiv 0\pmod 4\),则 \(n^2\equiv 0或1 \pmod 4\)

\((3)\) 由费马小定理,若 \((n,9)=1,n^{\varphi(9)}\equiv n^6\equiv 1\pmod 9\),即此时 \(n^{3}\equiv \pm 1\pmod 9\)

\((n,9)>1\),则 \(3\mid 9\),此时 \(n^3\equiv 0\pmod 9\),则 \(n^3\equiv 0,\pm 1\pmod 9\)

\((4)\) 由费马小定理,若 \((n,16)=1,n^{\varphi(16)}\equiv n^{8}\equiv 1\pmod {16}\),此时分解因式有 \[ (n^4-1)(n^4+1)\equiv 0\pmod{16} \]\(n^4\equiv 1\pmod{8}\),故 \(n^4\equiv 1,9\pmod{16}\) 综上有 \(n^4\equiv 0,1\pmod{16}\)

习题2.1.5

\(a\) 为奇数, \(n \geqslant 1 .\) 证明: \(a^{2^{n}} \equiv 1\left(\bmod 2^{n+2}\right)\)


证明:(使用归纳法,二项式展开较复杂)对 \(n=1\) 时成立(第一次作业第一题)

\(n=k\) 时成立,即 \(a^{2^{k}}\equiv 1\pmod {2^{k+2}}\)则存在 \(t\in Z ,a^{2^{k}}=1+2^{k+2}\cdot t\),则 \[ a^{2^{k+1}}\equiv(a^{2^{k}})^2\equiv(1+2^{k+2}t)^2\equiv1+2^{k+3}t+2^{2k+4}t^2\equiv 1\pmod{2^{k+3}} \] 由数学归纳法知,对 \(\forall \ n\geq 1\) 均成立

习题2.4.1

解下列同余方程:\((1)\) \(32 x \equiv 12(\bmod 8)\); \((2)\) \(28 x \equiv 124(\bmod 116)\); $ 5 x ()$


解:\((1)\) 由判定 \((32,8)=8\nmid 12\),则在 \(Z_{8}\) 下无解

\((2)\) 由判定 \((28,116)=4\mid 124\),则在 \(Z_{116}\) 下有 \(4\) 组解, \(7x\equiv 31\pmod{29}\),由费马小定理 \[ x\equiv 7^{\varphi(29)-1}\cdot 31\equiv 7^{27}\cdot 2\equiv 49^{13}\cdot 14\equiv -9^{13}\cdot 14\equiv 6^{6}\cdot 19\equiv 7^{3}\cdot 19\equiv 21\pmod {29} \] 则解为 \(x\equiv 21,50,79,108\)

\((3)\) 由判定 \((5,81)=1\mid 44\),则在 \(Z_{81}\) 下有 \(1\) 组解,有 \(5x\equiv 44\equiv 125\pmod{81}\)

\(x\equiv 25\pmod{81}\)

习题2.4.2

解下列同余方程组:

\((1)\) \(\left\{\begin{array}{l}x \equiv 1(\bmod 3), \\ x \equiv 1(\bmod 5), \\ x \equiv 2(\bmod 7) ;\end{array} \quad\right.\) \((2)\) \(\left\{\begin{array}{l}x \equiv 2(\bmod 4), \\ x \equiv 3(\bmod 5) \\ x \equiv 7(\bmod 9)\end{array}\right.\)


解:\((1)\) 由中国剩余定理,有 \(M_1=35,M_2=21,M_3=15\),而对应有 \[ 35M_1^{-1}\equiv 1\pmod{3},\ 21M_{2}^{-1}\equiv 1\pmod{5},\ 15M_{3}^{-1}\equiv 1\pmod{7} \]\(M_1^{-1}=2,M_{2}^{-1}=1,M_{3}^{-1}=1\),则 \(x\equiv 35\cdot 2\cdot 1+21\cdot 1\cdot 1+15\cdot 1\cdot 2\equiv 16\pmod{105}\)

\((2)\) 由中国剩余定理,有 \(M_1=45,M_2=36,M_3=20\),而对应有 \[ 45M_1^{-1}\equiv 1\pmod{4},\ 36M_{2}^{-1}\equiv 1\pmod{5},\ 20M_{3}^{-1}\equiv 1\pmod{9} \]\(M_1^{-1}=1,M_{2}^{-1}=1,M_{3}^{-1}=5\),则 \(x\equiv 45\cdot 1\cdot 2+36\cdot 1\cdot 3+20\cdot 5\cdot 7\equiv 178\pmod{180}\)

习题2.4.3

用中国剩余定理解同余方程 \(37 x \equiv 31(\bmod 77)\).


解:由 \(77=7\cdot 11\),对质因数 \(7\) 而言, \(37x\equiv 31\equiv185\pmod{7}\),则 \(x\equiv 5\pmod{7}\)

对质因数 \(11\) 而言,\(37x\equiv 31\equiv 185\pmod{11}\),则 \(x\equiv 5\pmod{11}\)

则由中国剩余定理,显然 \(x\equiv 5\pmod{77}\),为原方程的解,且该解唯一

习题2.4.4

\(2^{400}\)\(319\) 除的余数.


解:\(319=11\cdot 29\),对质因数 \(11\)\(2^{400}\equiv32^{80}\equiv (-1)^{80}\equiv 1\pmod{11}\),对质因数 \(29\) \[ 2^{400}\equiv 32^{80}\equiv 3^{80}\equiv 27^{26}\cdot 9\equiv 2^{26}\cdot 9\equiv 32^{5}\cdot 18\equiv 3^{5}\cdot 18\equiv 27^2\cdot 6\equiv 24\pmod{29} \] 则转化成一次同余方程组 \(\begin{cases}2^{400}\equiv 1\pmod{11}\\2^{400}\equiv 24\pmod{29}\end{cases}\),由中国剩余定理有

\(M_1=29,M_2=11,M_1^{-1}\cdot 29\equiv 1\pmod{11},M_2^{-1}\cdot 11\equiv 1\pmod{29}\)

解得 \(M_1^{-1}=8,M_{2}^{-1}=8\),则 \(2^{400}\equiv 29\cdot 8+11\cdot 8\cdot 24\equiv 2344\equiv 111\pmod{319}\)

习题2.4.5

\(m_{1}, m_{2}\) 是正整数, \(b_{1}, b_{2} \in \mathbf{Z}\). 证明: 同余方程组 \[ \left\{\begin{array}{l}x \equiv b_{1}\left(\bmod m_{1}\right), \\x \equiv b_{2}\left(\bmod m_{2}\right)\end{array}\right. \] 有整数解的充分必要条件是 \(\left(m_{1}, m_{2}\right) \mid b_{1}-b_{2}\),且此条件成立时, 解为模 \(\left[m_{1}, m_{2}\right]\) 的一个同余类.


证明:\((1)\) 充要性:若 \(\left\{\begin{array}{l}x \equiv b_{1}\left(\bmod m_{1}\right), \\x \equiv b_{2}\left(\bmod m_{2}\right)\end{array}\right.\) 有整数解,则 \(x=m_1y_1+b_1=m_2y_2+b_2\)

两边除以 \((m_1,m_2)\) 得到 \(\dfrac{b_1-b_2}{(m_1,m_2)}=\dfrac{m_2y_2-m_1y_1}{(m_1,m_2)}\in Z\),此时有 \((m_1,m_2)\mid b_1-b_2\)

\((m_1,m_2)\mid b_1-b_2\),有 \(\dfrac{m_2y_2-m_1y_1}{(m_1,m_2)}=\dfrac{b_1-b_2}{(m_1,m_2)}\in Z\),而 \((\dfrac{m_1}{(m_1,m_2)},\dfrac{m_2}{(m_1,m_2)})=1\)

由辗转相除法知,一定存在 \(y_{10},y_{20}\),使得 \(\dfrac{-m_1}{(m_1,m_2)}y_{10}+\dfrac{m_2}{(m_1,m_2)}y_{20}=1\)

则扩大 \(\dfrac{b_1-b_2}{(m_1,m_2)}\) 倍得到 \(y_1,y_2\) 的解,从而原同余方程组有解

\((2)\) 同余类:若上述条件成立时,对 \(\dfrac{-m_1}{(m_1,m_2)}y_{10}+\dfrac{m_2}{(m_1,m_2)}y_{20}=1\)\(y_{10}\) 对应解

所在同余类为模 \(\dfrac{m_2}{(m_1,m_2)}\) ,而 \(x=m_1y_1+b_1\) 则解为模 \(\dfrac{m_1m_2}{(m_1,m_2)}=[m_1,m_2]\)

习题2.4.6

\(m_{1}\)\(m_{2}\) 是互素的正整数. 证明:

\((1)\)\(S_{1}\)\(S_{2}\) 分别是模 \(m_{1}\) 和模 \(m_{2}\) 的完系. 则 \[ S=\left\{m_{1} x_{1}+m_{2} x_{2} \mid x_{1} \in S_{2}, x_{2} \in S_{1}\right\} \] 是模 \(m_{1} m_{2}\) 的完系;

  1. \(S_{1}\)\(S_{2}\) 分别是模 \(m_{1}\) 和模 \(m_{2}\) 的缩系, 则 \[ S=\left\{m_{1} x_{1}+m_{2} x_{2} \mid x_{1} \in S_{2}, x_{2} \in S_{1}\right\} \] 是模 \(m_{1} m_{2}\) 的缩系. (由此可知 \(\varphi\left(m_{1} m_{2}\right)=\varphi\left(m_{1}\right) \varphi\left(m_{2}\right)\) )

解:\((1)\)\(S\) 中的每个元素 \(x_1=m_2y_1+t_1\in S_2,x_2=m_1y_2+t_2\in S_1\)

\(m_1x_1+m_2x_2=m_1m_2(y_1+y_2)+t_1m_1+t_2m_2\),其中, \(y_1+y_2\in Z\) 能取遍所有整数

又由 \((m_1,m_2)=1\),由辗转相除法知 \(\exists\ t_{10},t_{20}\in Z\),使得 \(t_{10}m_1+t_{20}m_2=1\)

从而 \(t_1m_1+t_2m_2\) 也能取遍 \(0,1,\cdots ,m_1m_2-1\),则 \(S\) 为模 \(m_1m_2\) 的完系

\((2)\)\(\small(1)\) 相比,对 \(t_1m_1+t_2m_2\)\(t_1,t_2\) 不是任意整数,需满足 \((t_1,m_2)=1,(t_2,m_1)=1\)

此时 \(t_1m_1+t_2m_2\nmid m_1m_2\),且由辗转相除法知,存在 \(t_{10},t_{20}\) 使得 \(t_{10}m_1+t_{20}m_2=1\)

且反证法若 \(d=(t_{10},m_2)>1\),则上式对 \(d\) 同余有 \(1\equiv 0\pmod{d}\),矛盾,\(t_{10},t_{20}\) 满足条件

\(\forall \ 0\leq x\leq m_1m_2-1,(x,m_1m_2)=1\) \((x,m_1)=(x,m_2)=1\),取 \(xt_{10},xt_{20}\) 满足条件

从而 \(S\)\(m_1m_2\) 的缩系

习题2.4.7

\(n\) 为正整数,证明:必有连续 \(n\) 个正整数,其中每个整数均被某个大于 \(1\) 的整数的平方所除尽。


证明:构造一次同余方程 \(\begin{cases}x\equiv 0\pmod{2^2}\\x\equiv -1\pmod{3^2}\\x\equiv -2\pmod{5^2}\\\qquad\vdots\\x\equiv -(n-1)\pmod{p_{n}^2}\end{cases}\),其中 \(p_n\) 为第 \(n\) 个素数

\(\forall \ p_i,p_j,(p_i^{2},p_j^2)=1\),由中国剩余定理,一定有最小正整数解 \(x_{min}(n)\) 满足上述方程

\(x_{min}\) 能保证序列 \(x_{min},x_{min}+1,\cdots,x_{min}+(n-1)\) 中任何数能被某个大于 \(1\) 的整数平方整除

\(\large\textcolor{blue}{初等数论第五次作业}\ \ \ \ \ \ _\textcolor{blue}{2022.3.22}\)

习题3.1.2

\(a\) 对模 \(m\) 和模 \(n\) 的阶分别为 \(s\)\(t\), 证明: \(a\) 对模 \([m, n]\) 的阶为 \([s, t]\).


证明:由 \(a\) 对模 \(m\) 和 模 \(n\) 的阶为 \(s,t\),则 \(a^{s}\equiv 1\pmod m,a^{t}\equiv 1\pmod n\)

则令 \(a\)\([m,n]\) 的阶为 \(r\),即 \(a^{r}\equiv 1\pmod{[m,n]}\),有 \(m,n\mid [m,n]\),则 \(s,t\mid r\),则 \([s,t]\mid r\)

\(a^{[s,t]}\equiv (a^{s})^{\frac{[s,t]}{s}}\equiv 1\pmod{m},a^{[s,t]}\equiv (a^{t})^{\frac{[s,t]}{t}}\equiv 1\pmod{n}\),则 \(a^{[s,t]}\equiv 1\mod{[m,n]}\)

从而 \(r\mid [s,t]\),则 \(r=[s,t]\),即 \(a\) 对模 \([m,n]\) 的阶为 \([s,t]\)

习题3.1.5

\(n\)\(a\) 均是正整数, \(a \geqslant 2\). 证明: \(n \mid \varphi\left(a^{n}-1\right)\).


证明:考虑 \(a\)\(a^{n}-1\) 的阶 \(r\),有 \(a^{n}\equiv 1\pmod{a^{n}-1}\),则 \(r\mid n\),若 \(r=\dfrac{n}{d},d\geq 2\)

\(a^{r}=\sqrt[d]{a^{n}}\leq a^{n}-1\),则此时 \(a^{r}\not\equiv 1\pmod{a^{n}-1}\),从而 \(d=1\),即 \(a\)\(a^{n}-1\) 阶为 \(n\)

\(a\)\(a^{n}-1\) 阶一定为 \(\varphi(a^{n}-1)\) 的因子,从而 \(n\mid \varphi(a^{n}-1)\)

习题3.1.6

如果 \(n \geqslant 2\), 证明: \(n \not\mid 2^{n}-1\).


证明:由于 \(2^n-1\equiv 1\pmod{2}\),若 \(2\mid n\),则 \(n\not\mid 2^{n}-1\),若 \(2\not\mid n\),令 \(n\) 最小奇素数因子 \(p\)

\(n=pt,t\in Z\),反证法若 \(n\mid 2^n-1\),令 \(2^n=1+nx=2^{pt}=1+ptx,x\in Z\)

\(2^{pt}\equiv 1\pmod{p}\),而费马小定理给出 \(2^{p-1}\equiv 1\pmod{p},2^p\equiv 2\pmod{p}\)\(pt\mid p-1\) 矛盾

习题3.1.7

\(p\) 是奇素数, \(n \geqslant 1\). 证明: \[ \sum_{k=1}^{p-1} k^{n} \equiv \begin{cases}-1(\bmod p), & \text { 如果 } p-1 \mid n, \\ 0(\bmod p), & \text { 如果 } p-1 \not\mid n .\end{cases} \]


证明:由于 \(p\) 为奇素数,令其原根为 \(g\),则 \(Z_{p}=\{1,2,\cdots ,p-1\}=\{g,g^2,\cdots,g^{p-1}\}\)

从而求和式 \(\displaystyle \sum_{k=1}^{p-1}k^{n}\equiv \sum_{k=1}^{p-1}g^{n}\pmod{p}\),若 \(p-1\mid n\),由费马小定理 \(g^{n}\equiv g^{p-1}\equiv 1\pmod{p}\)

此时 \(\displaystyle \sum_{k=1}^{p-1}k^{n}\equiv \sum_{k=1}^{p-1}1\equiv -1\pmod{p}\),若 \(p-1\not\mid n\),则由等比数列求和 \[ \sum_{k=1}^{p-1}g^n\equiv \dfrac{g^{n}(1-g^{p-1})}{1-g^{n}}\equiv \dfrac{g^{n}\cdot 0}{1-g^{n}}(1-g^{n}\neq 0)\equiv 0\pmod{p} \] 从而 \(\sum_{k=1}^{p-1} k^{n} \equiv \begin{cases}-1(\bmod p), & \text { if } p-1 \mid n, \\ 0(\bmod p), & \text { if } p-1 \not\mid n .\end{cases}\)

习题3.1.8

\((1)\)\(F_{n}=2^{2^{n}}+1\) (费马数), \(n \geqslant 1\). 证明: \(F_{n}\) 的每个素因子都有形式 \(2^{n+1} x+1(x \in \mathbf{Z})\).

\((2)\) 对任意给定的整数 \(l \geqslant 1\), 证明: 有无穷多个素数模 \(2^{l}\) 余 1 .


证明:\((1)\) 对每个 \(F_{n}\) 的奇素数因子 \(p\),有 \(2^{p-1}\equiv 1\pmod{p}\),而 \(2^{2^{n}}\equiv -1\pmod{p}\),两边平方

\(2^{2^{n+1}}\equiv 1\pmod{p}\),从而令 \(2\)\(p\) 的次数为 \(r\),则 \(r\mid2^{n+1}\)\(r=2^{k}\),显然 \(k\neq n\),因为

\(2^{2^{n}}\equiv -1\pmod{p}\),若 \(k<n\),则重复 \(n-k\) 次平方操作得到 \(2^{2^{n}}\equiv 1\pmod{p}\) 与条件矛盾

从而 \(r=2^{n+1}\),结合 \(2^{p-1}\equiv 1\pmod{p}\) 可得 \(2^{n+1}\mid p-1\)

\((2)\) 对任意两个不相同的费马数,\(F_{m},F_{n}\) ,不妨设 \(m\geq n,m=n+k,k\geq 1\in Z\)

\(F_{m}=2^{2^{m+n}}+1=(2^{2^{n}})^{2^k}+1=(F_{n}-1)^{2^k}+1=p(F_n)+2\),其中 \(p(F_n)\) 为多项式

从而 \((F_{m},F_n)\mid 2\),又 \(x\not\mid F_{n},F_m\),则 \((F_m,F_n)=1\),即所有费马数两两互素,而每个 \(F_n\)

都至少有一个素因子模 \(2^{n+1}\)\(1\),且不同费马数素因子不重合,则对给定的 \(l\),取 \(n\geq l-1\)

则这些无穷个的费马数模 \(2^{n+1}\)\(1\),则也模 \(2^{l}\)\(1\)

习题3.1.9

\((1)\)\(p\) 为奇素数, \(a \geqslant 2\). 证明: 若 \(a^{p}-1\) 的素因子 \(q\) 不整除 \(a-1\),则必有形式 \(q=2 p x+1(x \in Z)\).

\((2)\)\(p\) 为给定的奇素数, 证明:形如 \(2 p x+1(x \in \mathbb{Z})\) 的素数有无限多个.


证明:\((1)\) 由于 \(q\)\(a^{p}-1\) 素因子,则 \(a^p\equiv 1\pmod{q}\),由 \(q\not\mid a-1\),得 \(a\not\equiv 1\pmod{q}\)

\(a\) 为模 \(q\) 的阶为 \(r\),则有 \(r\mid p\),又 \(r\neq 1\),从而 \(r=p\),有费马小定理 \(a^{q-1}\equiv 1\pmod{q}\)

从而 \(r=p\mid q-1\);另一方面考虑奇偶性,若 \(2 \mid a\)\(2\not \mid a^{p}-1\),素因子 \(q\) 为奇数,若 \(2\not\mid a\)

\(a^{p}-1\) 有素因子 \(2\),但此时要求 \(q\) 不整除 \(a-1\),从而 \(q\neq 2\),素因子 \(q\) 也为奇数,则 \(q-1\)

有素因子 \(2\),而 \(p\) 为奇素数,从而 \(2p \mid q-1\)

\((2)\) 构造 \(a_1=2^{p}-1,a_n=(\displaystyle \prod_{i=1}^{n-1}a_i)^{p}-1\),满足两两互素,\(a^{p}-1\) 总能有不整除 \(a-1\) 的素因子

否则所有素因子 \(p_i=\dfrac{a-1}{k_i},k_i\geq 2\in Z\),相乘有 \(a^p-1=\dfrac{(a-1)^{x}}{\displaystyle \prod_{i=1}^{x}k_i}\) 移项后有

\((a^p-1)\displaystyle \prod _{i=1}^{x}k_i=(a-1)^x\),两边对 \(a\) 取模得 \(-\displaystyle \prod_{i=1}^xk_i\equiv (-1)^x\pmod{a}\) 矛盾

则两两互素的无穷整数数列 \(\{a_n\}\) 中每个数都存在形如 \(2px+1\) 的素因子,共无穷多个

柯召讲义5.1

证明: \(m\) 是一个素数的充分必要条件是存在某个整数 \(a, a\) 对模数 \(m\) 的次数为 \(m-1\).


证明:\(1^{\circ}\)\(m\) 为一个素数,由素数必有原根可知,\(\exists\ a\in Z\),使得 \(a\)\(m\) 的阶数为 \(m-1\)

\(2^{\circ}\) 若对某个 \(a\in Z\)\(a^{r}\equiv 1\pmod{m}\) 的最小正整数(阶数 \(r\) )为 \(m-1\),有 \((a,m)=1\)

由费马小定理 \(a^{\varphi(m)}\equiv 1\pmod{m}\),得 \(m-1\mid \varphi(m)\),若 \(m\) 不是素数,\(m\geq 2\),令 \[ m=p_1^{l_1}\cdots p_{s}^{l_s},\varphi(m)=m\prod _{i=1}^{s}(1-\dfrac{1}{p_i})\leq m\prod _{i=1}^{s}(1-\dfrac{1}{p_i^{l_i}})< m(1-\prod_{i=1}^{s}\dfrac{1}{p_i^{l_i}})=m-1 \] 从而 \(m\) 为素数

柯召讲义5.2

\(g\) 是奇素数 \(p\) 的一个原根, 证明: 当 \(p \equiv 1(\bmod 4)\) 时, \(-g\) 也是 \(p\) 的一个原根; 当 \(p \equiv 3(\bmod 4)\) 时, \(-g\)\(p\) 的次数为 \(\dfrac{p-1}{2}\).


证明:若 \(g\) 是奇素数 \(p\) 的一个原根,则满足 \(g^{r}\equiv 1\pmod{p}\) 的最小 \(r=p-1\),而 \(2\mid p-1\)

\((-g)^{p-1}\equiv g^{p-1}\equiv 1\pmod{p}\),且若存在 \(1\leq r'< p-1\),使得 \((-g)^{r'}\equiv 1\pmod{p}\)

\(r'\) 不能是偶数,否则与阶数矛盾,则 \(2\not \mid p\),有 \(g^{r'}\equiv -1\pmod{p}\),则 \[ g^{2r'}-1\equiv (g^{r'}+1)\cdot (g^{r'}-1)\equiv 0\pmod{p},g^{2r'}\equiv 1\pmod{p} \]\(2r'\mid r\),由于 \(r'\) 为奇数,若 \(p\equiv 1\pmod{4}\),则 \(4\mid p-1\),此时 \(r'\) 不存在,从而 \(-g\) 也为原根

\(p\equiv 3\pmod{4}\) ,有 \[ g^{p-1}-1\equiv (g^{\frac{p-1}{2}}-1)\cdot (g^{\frac{p-1}{2}}+1)\equiv 0\pmod{p} \]\(g^{\frac{p-1}{2}}\not \equiv 1\pmod{p}\) 否则不满足阶数的性质,则 \((-g)^{\frac{p-1}{2}}\equiv 1\pmod{p}\),若 \(1\leq r''< \dfrac{p-1}{2}\)

使得 \((-g)^{r''}\equiv 1\pmod{p}\),首先 \(r''\) 不为偶数,若 \(r''\) 为奇数,则 \(g^{r''}\equiv -1\pmod{p}\)

从而平方 \(g^{2r''}\equiv 1\pmod{p}\),则有 $22r''<p-1 $ 使得 \(g^{2r''}\equiv 1\pmod{p}\) 与阶数矛盾

从而此时 \(-g\)\(p\) 的阶数为 \(\dfrac{p-1}{2}\)

\(\large\textcolor{blue}{初等数论第六次作业}\ \ \ \ \ \ _\textcolor{blue}{2022.4.1}\)

习题3.1.1

\(m \geqslant 2\), 整数 \(a\)\(b\)\(m\) 的阶分别为 \(s\)\(t\), 并且 \((s, t)=1\). 证明 \(a b\)\(m\) 的阶为 \(s t\).


证明:设 \(ab\)\(m\) 的阶为 \(r\)\((ab)^{r}\equiv 1\pmod{m}\),由 \((ab)^{st}\equiv (a^{s})^{r}\cdot (b^r)^{s}\equiv 1\pmod{m}\)

\(r\mid st\),则设 \(st=pr,p\in Z \mbox{\ and \ p is\ prime}\),由 \((s,t)=1\),则 \(p\mid s\)\(p \mid t\),不妨设 $ ps$

\(s=px,xt=r\),有 \((ab)^{r}\equiv (ab)^{xt}\equiv a^{xt}\cdot 1\equiv 1 \pmod{m}\)

\(s\mid xt\),由 \((s,t)=1\),得 \(s\mid x\),又 \(x\mid s\),可得 \(s=x,p=1\),即 \(r=st\)

习题3.1.4

\((1)\) 对于 \(p=5,7,11,13,23\), 求模 \(p\) 的最小正原根.

\((2)\) 求模 \(7^{2}\) 的全部原根.


解:\((1)\) 由在 \(Z_5\)\(\{2,2^2,2^3,2^4\}=\{2,4,3,1\}\) ,则 \(2\)\(5\) 的最小正原根

\(Z_7\)\(\{2,2^2,2^3,2^4,2^5,2^6\}=\{2,4,1,2,4,1\},\{3,3^2,3^3,3^4,3^5,3^6\}=\{3,2,6,4,5,1\}\)

\(3\)\(7\) 的最小正原根

\(Z_{11}\)\(\{2,2^2,2^3,2^4,2^5,2^6,2^7,2^8,2^9,2^{10}\}=\{2,4,8,5,10,9,7,3,6,1\}\) ,则 \(2\)\(11\)

最小正原根

\(Z_{13}\)\(\{2,2^2,2^3,2^4,2^5,2^6,2^7,2^8,2^9,2^{10},2^{11},2^{12}\}=\{2,4,8,3,6,12,11,9,5,10,7,1\}\)

\(2\)\(13\) 的最小正原根

\(Z_{23}\)\(\{2,2^2,2^3,2^4,2^5,2^6,2^7,2^8,2^9,2^{10},2^{11}\}=\{2,4,8,16,9,18,13,3,6,12,1\}\)

\(\{3,3^2,3^3,3^4\}=\{3,6,12,1\}\)\(5\) 而言 \[ \begin{gathered} \{5,5^2,5^3,5^4,5^5,5^6,5^7,5^8,5^9,5^{10},5^{11},5^{12},5^{13},5^{14},5^{15},5^{16},5^{17},5^{18},5^{19},5^{20},5^{21},5^{22}\}=\\ \{5,2,10,4,20,8,17,16,11,9,22,18,21,13,19,3,15,6,7,12,14,1\} \end{gathered} \]\(5\)\(23\) 的最小正原根

\((2)\) 由定理, \(7\) 有原根 \(3\),则 \(3\)\(10\) 必有一个为 \(49\) 的原根,有 \[ 3^{2},3^{4},3^{6},3^{8},3^{12},3^{16},3^{24}\equiv 9,32,43,44,36,25,22\pmod{49} \] 从而 \(3\)\(49\) 的原根,有 \(\varphi(\varphi(49))=\varphi(42)=12\) 个原根,有 \(3^{r},(r,42)=1\) 均为原根,计算有

\(\{3,3^{5},3^{11},3^{13},3^{17},3^{19},3^{23},3^{25},3^{29},3^{31},3^{37},3^{41}\}=\{3,47,12,10,26,38,40,17,5,45,24,33\}\)

\(49\) 的所有原根为 \(\begin{equation}\boxed{3, 5, 10 , 12, 17 , 24 , 26 , 33 , 38 , 40 , 45, 47}\end{equation}\),共 \(12\)

习题3.2.1

解同余方程:

\((1)\) \(x^{8} \equiv 3 \pmod{13}\);

\((2)\) \(x^{8} \equiv 3\pmod {143}\);

\((3)\) \(7^{x} \equiv 4\pmod{17}\).


解:\((1)\) 由上题知 \(2\)\(13\) 的一个原根,则原式等价于 \(8\mbox{ind}_2x\equiv \mbox{ind}_{2}3\pmod{12}\)

\(2^{4}\equiv 3\pmod{13}\)\(8\mbox{ind}_2x\equiv 4\pmod{12}\)\((8,12)=4\mid 4\),代换有

\(2\mbox{ind}_2x\equiv 1\equiv 4\pmod{3}\)\(\mbox{ind}_2{x}\equiv 2,5,8,11\),得解为 \(x_1\equiv 4,6,7,9\pmod{13}\)

\((2)\) 结合 \(x^8\equiv 3\pmod{11}\),由 \(2\)\(11\) 的一个原根,等价于 \(8\mbox{ind}_2x\equiv \mbox{ind}_23\equiv 8\pmod{10}\)

\((8,10)=2\mid 10\),则有两个解,\(\mbox{ind}_2x\equiv 1,6\pmod{10}\),则 \(x_2\equiv 2,9\pmod{11}\)

使用中国剩余定理,\(11^{-1}\equiv 6\pmod{13},13^{-1}\equiv 6\pmod{11}\) 对应计算 \(x_1\cdot 6\cdot 13+x_2\cdot 6\cdot 11\) \[ x\equiv 9,20,35,46,97,108,123,134 \pmod{143} \] \((3)\)\(2\) 计算 \(2^1,2^2,2^4,2^8\equiv 2,4,16,1\pmod{17}\),则 \(2\) 不是 \(17\) 的原根,同理对 \(3\) 计算有

\(3^1,3^2,3^4,3^8\equiv 3,9,13,16\pmod{17}\),则 \(3\)\(17\) 的原根,则对 \(7^{x} \equiv 4\pmod{17}\) 变换有

\(x\mbox{ind}_{3}7\equiv \mbox{ind}_34\pmod{16}\)\(\mbox{ind}_37=11,\mbox{ind}_34=12\),则 \(11x\equiv 12\pmod{16}\)

\((11,16)=1\mid 12\) 只有一个整数解,得 \(x\equiv 4\pmod{17}\)

习题3.2.2

\((1)\) 写出模 \(37\) 的全部 \(8\) 次剩余和 \(15\) 次剩余.

\((2)\) 写出模 \(11\) 的全部二次剩余.


解:\((1)\)\(37\) 的一个原根 \(2\),由 \((8,36)=4\),因此八次剩余 \(a\) 满足的条件为: \(4 \mid \operatorname{ind}_{2} a\) ,故

\(\operatorname{ind}_{2} a=4,8,12, \cdots, 36\) ,因此模 \(37\) 的全部 \(8\) 次剩余为: \(a \equiv 2^{4}, 2^{8}, \cdots, 2^{36}(\bmod 37)\)\[ a\equiv 16,34,26,9,33,10,12,7,1\pmod{37} \]\((15,36)=3\) ,因此 \(15\) 剩余 \(a\) 满足的条件为:\(3 \operatorname{ind}_{2} a\) ,故 \(\operatorname{ind}_{2} a=3,6, \cdots, 36\) ,因此解为:

\[ a \equiv 2^{3}, 2^{6}, \cdots, 2^{36}\equiv 8,27,31,26,23,36,29,10,6,11,14,1 (\bmod 37) \] \((2)\)\(11\) 的全部二次剩余为: \(1^{2}, 2^{2}, \cdots, 5^{2}\) ,即 \(1,3,4,5,9\)

习题3.2.3

\(p\) 为素数, \(p \equiv 2(\bmod 3), a\)\(b\) 是整数. 证明: \(a^{3} \equiv b^{3}(\bmod p)\) 当且仅当 \(a \equiv b(\bmod p)\).


证明:若 \((a,p)=p\),则 \(a\equiv b\equiv 0\pmod{p}\),否则 \((a,p)=1\)\(p=3k+2,k\geq 1\),有

\(a^{3}\equiv b^{3}\equiv c\pmod{p}\),且 \((c,p)=1\),由欧拉定理 \(a^{3k+1}\equiv b^{3k+1}\equiv c^ka\equiv c^kb\equiv 1\pmod{p}\)

从而 \(a\equiv c^{-k}\equiv b\pmod{p}\)

\(a^{3} \equiv b^{3} \equiv k(\bmod p)\) ,因为 \((3, p-1)=1\) ,故 \(x^{3} \equiv k(\bmod p)\) 在模 \(p\) 意义下仅有一个解, 故得证

习题3.2.4

\(p\) 为奇素数, 整数 \(a\)\(p\) 的阶为 \(3\),求 \(\dfrac{a}{a+1}\)\(p\) 的阶.


解:由题设定义 \(a\) 的数论倒数为 \(b\),即 \(ab\equiv 1\pmod{p}\),有 \((\dfrac{a}{a+1})^{-1}=1+\dfrac{1}{a}=1+b\)

\(x\)\(x^{-1}\)\(p\) 的阶是一样的,从而考虑 \(1+b\)\(p\) 的阶,由满足 \(a^{r}\equiv 1\pmod{p}\) 的最小正整数

\(3\) ,也有 \(b^3\equiv 1\pmod{p}\)\(b^2+b+1\equiv 0\pmod{p}\),令 \(1+b\)\(p\) 的阶为 \(r'\)

\((1+b)^{r'}\equiv 1\pmod{p}\) ,有 \((1+b)^{1}\not\equiv 1\pmod{p},(1+b)^2\equiv b\not\equiv 1\pmod{p}\)

\((1+b)^3\equiv 3(b^2+b+1)-2+b^3\equiv -1\pmod{p}\),而 \((1+b)^4\equiv -1\not\equiv 1\pmod{p}\)

\((1+b)^5\equiv -b\not\equiv 1\pmod{p}\),且 \((1+b)^{6}\equiv 1\pmod{p}\),则 \(\dfrac{a}{a+1}\)\(p\) 的阶为 \(6\)

习题4.1.2

\(p\) 为奇素数, \(n\) 是最小正整数使得 \(\left(\dfrac{n}{p}\right)=-1\). 证明: \(n\) 为素数


证明:由勒让德符号定义不存在整数 \(x\) 使得 \(x^2\equiv n\pmod{p}\),取 \(p\) 的原根 \(g\)\(n=g^{2j+1}\)

\(n\) 为合数,即 \(n=n_1n_2\),由 \(\left(\dfrac{n}{p}\right)=\left(\dfrac{n_1}{p}\right)\left(\dfrac{n_2}{p}\right)\) 可知 \(n_1\)\(n_2\) 一定一者为 \(1\),另一者为 \(-1\)

而显然为 \(-1\) 的一者对应的 \(n_i\) 更小,与题设矛盾,从而 \(n\) 为素数

\(\large\textcolor{blue}{初等数论第七次作业}\ \ \ \ \ \ _\textcolor{blue}{2022.4.16}\)

习题4.1.1

\(p\) 为奇素数, \(a, b \in \mathbf{Z},(a, p)=1\). 证明: \[ \sum_{n=0}^{p-1}\left(\frac{a n+b}{p}\right)=0 \]


证明:由于 \((a,p)=1\),则 \(a,a+b,\cdots,a(p-1)+b\) 构成模 \(p\) 的完系,可用反证法证明

其中模 \(p\) 的二次剩余和非二次剩余都为 \(\dfrac{p-1}{2}\) 个,最后剩一个能被 \(p\) 整除的

\(\displaystyle \sum_{n=0}^{p-1}\left(\frac{a n+b}{p}\right)=\dfrac{p-1}{2}+(-1)\cdot \dfrac{p-1}{2}+0=0\)

习题4.1.3

证明形如 \(8m+3,8m+5,8m+7\) 的素数均有无穷多个


解:\((1)\) 反证,假设 \(8m+3\) 的素数有限个,设为 \(p_1.p_2,\cdots ,p_s\) 构造 \(n=2(p_1p_2\cdots p_s)^2+1\)

\(n\equiv 2\cdot (3^2)^{s}+1\equiv 3\pmod{8}\),则由假设 \(n\) 为合数

设其素因子为 \(p\),有 \(2(p_1p_2\cdots p_s)^2\equiv -1\pmod{p}\),两边乘以 \(2\),取勒让德符号 \[ (\dfrac{-2}{p})=1=(\dfrac{2}{p})(\dfrac{-1}{p})=(-1)^{\frac{p^2-1}{8}+\frac{p-1}{2}}=(-1)^{\frac{(p-1)(p+5)}{8}} \] 从而 \(2\mid \dfrac{(p-1)(p+5)}{8}\),从而 \(p\equiv 1,3\pmod{8}\)。但 \(n\equiv 3\pmod{8}\)\(n\) 的所有素因子不可能

均模 \(8\)\(3\),从而一定存在一个模 \(8\)\(3\) 的素因子,而该素因子不为 \(p_1,\cdots ,p_s\),这是因为

\(n\equiv 1\pmod{p_i}\),从而矛盾,\(8m+3\) 型素数有无穷多个

\((2)\) 假设 \(8m+5\) 的素数有限个,设为 \(p_1,p_2,\cdots,p_s\),构造 \(n=4(p_1\cdots p_s)^2+1\)

\(n\equiv 5\pmod{8}\),其为合数,设其素因子为 \(p\)

\((2p_1\cdots p_s)^2\equiv -1\pmod{p}\),取勒让德符号有 \((\dfrac{-1}{p})=1=(-1)^{\frac{p-1}{2}}\)

\(p\equiv 1,5\pmod{8}\)\(n\equiv 5\pmod{5}\)\(n\) 的所有素因子不可能全为 \(1\),则其存在一个

不为 \(p_i\) 的素因子,与假设矛盾,从而 \(8m+5\) 型素数有无穷多个

\((3)\) 假设 \(8m+7\) 的素数有限个,设为 \(p_1,p_2,\cdots,p_s\),构造 \(n=8(p_1\cdots p_s)^2-1\)

\(n\equiv 7\pmod{8}\),其为合数,设其素因子为 \(p\)

\((4p_1\cdots p_s)^2\equiv 2\pmod{p}\),使用勒让德符号有 \((\dfrac{2}{p})=1=(-1)^{\frac{p^2-1}{8}}\)

\(p\equiv 1,7\),又由于 \(n\equiv 7\pmod{8}\)\(n\) 所有素因子不可能全为 \(1\),则其存在一个

不为 \(p_i\) 的素因子,与假设矛盾,从而 \(8m+7\) 型素数有无穷多个

习题4.1.4

\(p\) 为奇素数. 证明: 有无穷多的素数是模 \(p\) 的二次非剩余.


证明:假设满足是模 \(p\) 二次非剩余的素数有限,为 \(p_1,p_2,\cdots p_s\),有 \((\dfrac{p_i}{p})=-1\),取模 \(p\)

一个二次非剩余 \(a\),由中国剩余定理,满足 \(x\equiv a\pmod{p},x\equiv 1\pmod{p_i}\) 同余方程的解

存在,设其中一个解为 \(x\)

\((\dfrac{x}{p})=(\dfrac{a}{p})=-1\)\(x\) 必有素因子为二次非剩余,且使得该素因子与 \(p_i\) 均互素,这样与

有限个满足条件的素数矛盾,从而有无穷多个素数模 \(p\) 二次剩余

习题4.1.5

\(p\)\(q=2 p+1\) 都是素数. 证明:

\((1)\)\(p \equiv 1(\bmod 4)\) 时, \(2\) 是模 \(q\) 的原根;

\((2)\)\(p \equiv 3(\bmod 4)\) 时,\(-2\) 是模 \(q\) 的原根.


解:\((1)\) 由于 \(q-1=2p\) 只有素因子 \(2\)\(p\) ,有 \(2^2\equiv 4\not\equiv 1\pmod{q}\) 而对\(2^{p}\equiv 2^{\frac{q-1}{2}}\),计算

\((\dfrac{2}{q})=(-1)^{\frac{q^2-1}{8}}=(-1)^{\frac{p(p+1)}{2}}=-1\),则 \(2\) 为模 \(q\) 的二次非剩余,则取 \(q\) 的原根 \(g\),有 \(2=g^{2j+1}\),代入计算 \(2^{p}=g^{(2j+1)p}=(g^{p})^{2j+1}=(g^{\frac{q-1}{2}})^{2j+1}=(-1)^{2j+1}=-1\),从而 \(2,p\) 均不是模 \(p\)

\(2\) 的阶数,从而 \(2\) 为模 $p $ 的原根

\((2)\) 同上,只需考虑 \((-2)^{p}\)\(p\neq 2\),计算 \((\dfrac{-2}{q})=(\dfrac{-1}{q})(\dfrac{2}{q})=(-1)^{\frac{q-1}{2}}(-1)^{\frac{q^2-1}{8}}=-1\)

则取 \(q\) 的原根 \(g\),有 \(-2=g^{2j+1}\),代入 \((-2)^{p}=(g^{\frac{q-1}{2}})^{2j}=(-1)^{2j+1}=-1\)

从而 \(2,p\) 均不是模 \(p\)\(-2\) 的阶数,从而 \(-2\) 为模 $p $ 的原根

习题4.1.6

\(p\) 为素数, \(p \equiv 3(\bmod 4)\). 记 \(q=2 p+1\). 则 \(q\) 为素数当且仅当 \(q \mid 2^{p}-1\).


证明:由上题知,若 \(q\) 为素数,\(-2\)\(q\) 的原根 \[ 2^{p}\equiv -(-2)^{p}\equiv -(-2)^{\frac{q-1}{2}}\equiv -(-1)\equiv 1\pmod{p} \]\(q\mid 2^{p}-1\) 。又若 \(q\mid 2^{p}-1\) ,由欧拉定理 \(2^{\varphi(q)}\equiv 1\pmod{q}\),即 \(q\mid 2^{\varphi(q)}-1\)

\(p\mid \varphi(q)=\varphi(2p+1)\),若 \(q\) 不是素数,\(\varphi(2p+1)<2p\),则 \(p=\varphi(2p+1)\),而对于奇数

的欧拉函数为偶数,从而矛盾,则 \(q\) 为素数

习题4.1.7

\(p\) 为奇素数. 证明: \[ \large \prod_{\substack{r=1 \\ \\ \\ \large (\frac{r}{p})=1}}^{p-1} \normalsize r \equiv-\left(\frac{-1}{p}\right)(\bmod p) \]


解:满足 \((\dfrac{r}{p})=1\) 的所有 \(r\)\(g^0,g^2,\cdots ,g^{p-3}\)。其中 \(g\)\(p\) 的一个原根,则 \[ \displaystyle \prod r =g^{0+2+\cdots p-3}=g^{\frac{(p-3)(p-1)}{4}}=(g^{\frac{p-1}{2}})^{\frac{p-3}{2}}=(-1)^{\frac{p-3}{2}} \]\(-(\dfrac{-1}{p})=-(-1)^{\frac{p-1}{2}}=(-1)^{\frac{p-3}{2}}\),两者相等

习题4.1.8

\(p\) 为素数,

\((1)\)\(p \equiv 1(\bmod 4)\), 则 \[ \sum_{\substack{r=1 \\\left(\frac{r}{p}\right)=1}}^{p-1} r=\frac{p(p-1)}{4}, \quad \sum_{a=1}^{p-1} a\left(\frac{a}{p}\right)=0 \] (2) 若 \(p \equiv 3(\bmod 4)\), 并且 \(p \geqslant 7\), 则 \[ \sum_{\substack{r=1 \\\left(\frac{r}{p}\right)=1}}^{p-1} r \equiv 0(\bmod p), \sum_{r=1}^{p-1} a\left(\frac{a}{p}\right) \equiv 0(\bmod p) . \]


证明:\((1)\)\((\dfrac{p-x}{p})=(\dfrac{-x}{p})=(\dfrac{x}{p})(\dfrac{-1}{p})=(\dfrac{x}{p})(-1)^{\frac{p-1}{2}}=(\dfrac{x}{p})\)

从而 \((i,p-i)\) 每对勒让德符号相同,从而所有满足 \((\dfrac{r}{p})=1\)\(r\) 一定能配成和为 \(p\)\(\mbox{pairs}\)

且共有 \(\dfrac{p-1}{4}\) 对,求和为 \(\dfrac{p(p-1)}{4}\),同理分离得到 \(\displaystyle \sum_{a=1}^{p-1}a(\dfrac{a}{p})=0+\sum_{\substack{r=1 \\\left(\frac{r}{p}\right)=1}}^{p-1} r-\sum_{\substack{r=1 \\\left(\frac{r}{p}\right)\neq 1}}^{p-1} r=0\)

\((2)\) 和上题类似,得到 \((\dfrac{p-x}{p})=-(\dfrac{x}{p})\),从而 \((i,p-i)\) 勒让德符号相反,有记 \[ \sum_{\substack{r=1 \\\left(\frac{r}{p}\right)=1}}^{p-1} r \equiv A(\bmod p),\sum_{\substack{r=1 \\\left(\frac{r}{p}\right)\neq 1}}^{p-1} r \equiv B(\bmod p) \] 取模 \(p\) 的一个原根 \(g\) ,则 \(g\) 是模 \(p\) 的一个二次非剩,\((\dfrac{g}{p})=-1\) \[ \begin{aligned} &\sum_{r=1,\left(\frac{r}{p}\right)=1}^{p-1} r \equiv 1+g^{2}+\cdots+g^{p-3} \equiv \frac{1-g^{\frac{p-1}{2}\cdot 2}}{1-g^{2}} \equiv 0 \quad(\bmod p) \\ &\sum_{a=1}^{p-1} a\left(\frac{a}{p}\right) \equiv \sum_{i=0}^{p-2} g^{i}\left(\frac{g^{i}}{p}\right) \equiv \sum_{i=0}^{p-2} g^{i}(-1)^{i} \equiv \sum_{i=0}^{p-1}(-g)^{i} \equiv \frac{1-g^{p-1}}{1+g} \equiv 0 \quad(\bmod p) . \end{aligned} \]

习题4.2.1

计算 \(\left(\dfrac{17}{23}\right),\left(\dfrac{19}{37}\right),\left(\dfrac{92}{101}\right)\).


由二次互反律计算有 \[ \left(\dfrac{17}{23}\right)=\left(\dfrac{23}{17}\right)(-1)^{11 \cdot 8}=\left(\dfrac{23}{17}\right)=\left(\dfrac{6}{17}\right)=\left(\dfrac{2}{17}\right)\left(\dfrac{3}{17}\right)\\=(-1)^{\frac{17^{2}-1}{8}}\left(\dfrac{17}{3}\right)(-1)^{8}=\left(\dfrac{17}{3}\right)=\left(\dfrac{2}{3}\right)=-1\\ \left(\dfrac{19}{37}\right)=\left(\dfrac{37}{19}\right)(-1)^{9 \cdot 18}=\left(\dfrac{37}{19}\right)=\left(\dfrac{-1}{19}\right)=(-1)^{9}=-1\\ \left(\dfrac{92}{101}\right)=\left(\dfrac{2}{101}\right)^{2}\left(\dfrac{23}{101}\right)=(-1)^{\dfrac{101^{2}-1}{8}}\left(\dfrac{23}{101}\right)=\\\left(\dfrac{23}{101}\right)=\left(\dfrac{101}{23}\right)(-1)^{50 \cdot 11}=\left(\dfrac{101}{23}\right)=\left(\dfrac{9}{23}\right)=\left(\dfrac{3}{23}\right)^{2}=1 \]

习题4.2.4

\(p=10 m-1\) 为素数, 证明: \(p \mid 5^{5 m-1}-1\).


证明:计算 \((\dfrac{5}{p})=(\dfrac{p}{5})(-1)^{2\cdot \frac{p-1}{2}}=(\dfrac{p}{5})=(\dfrac{-1}{5})=(-1)^2=1\),则 \(5\)\(p\) 的二次剩余

\(g\)\(p\) 的二次剩余,\(5\equiv g^{2j}\),有 \(5^{5m-1}\equiv (g^{2j})^{\frac{p-1}{2}}\equiv (g^{p-1})^{j}\equiv 1\pmod{p}\)

习题4.2.5

\(n \geqslant 2, p=2^{n}+1\) 为素数. 证明:

\((1)\) 对每个 \(a \in \mathbf{Z}, a\) 为模 \(p\) 的原根当且仅当 \(\left(\dfrac{a}{p}\right)=-1\);

\((2)\) 证明 \(3\)\(7\) 均为模 \(p\) 的原根.


证明:\((1)\)\(a\) 为模 \(p\) 的原根,有 \((\dfrac{a}{p})=-1\);若 \((\dfrac{a}{p})=-1\),若 \(a\) 不是模 \(p\) 的原根,则对

\(p-1=2^n\) 的因子为对应的阶,总有 \(a^{2}\equiv 1\pmod{p}\),这与 \((\dfrac{a}{p})=-1\) 矛盾,从而 \(a\) 是模 \(p\) 的原根

\((2)\) 由于 \(p=2^{n}+1\) 为素数,故 \(n=2^{k}\) ,其中 \(k \geq 1\).

\((\dfrac{3}{p})=(\dfrac{p}{3})(-1)^{1\cdot \frac{p-1}{2}}=(\dfrac{p}{3})\) ,而 \(2^{2^n}+1\equiv (-1)^{2^n}+1\equiv 2\pmod{3}\)

\((\dfrac{2}{3})=-1\),则 \((\dfrac{3}{p})=-1\),从而 \(3\) 为模 \(p\) 的原根;同理 \((\dfrac{7}{p})=(\dfrac{p}{7})\)

\(2^{k} \equiv 1(\bmod 3)\) ,则 \(\left(\dfrac{2^{2^{k}}+1}{7}\right)=\left(\dfrac{\left(2^{3}\right)^{t} \cdot 2+1}{7}\right)=\left(\dfrac{3}{7}\right)=-\left(\dfrac{7}{3}\right)=-1\).

\(2^{k} \equiv-1(\bmod 3)\) ,则 \(\left(\dfrac{2^{2^{k}}+1}{7}\right)=\left(\dfrac{\left(2^{3}\right)^{t} \cdot 2^{2}+1}{7}\right)=\left(\dfrac{5}{7}\right)=\left(\dfrac{7}{5}\right)=\left(\dfrac{2}{5}\right)=(-1)^{\frac{5^{2}-1}{8}}=-1\).

因此 \(\left(\dfrac{7}{p}\right)=-1\) ,故 \(7\) 是模 \(p\) 的原根.

\(\large\textcolor{blue}{初等数论第八次作业}\ \ \ \ \ \ _\textcolor{blue}{2022.4.23}\)

习题4.2.2

确定以 \(a\) 为二次剩余的素数, 其中 \(a=-3,5\)\(15\) .


\(a=-3\) 时,\(2\) 满足题意,此外, \(\dfrac{1}{2} \varphi(3)=1\). 且 \(7\) 满足题意,则解为奇素数 \(p=3 k+1\)\(2\)

\(a=5\equiv 1\pmod{4}\)\(2\) 满足题意. 此外, \(\dfrac{1}{2} \varphi(5)=2\) ,且 \(11,19\) 均满足题意. 故 \(2\) 和奇素数 \(5 k \pm 1\) 满足题意.

\(a=15\) 时, \(2\) 满足题意. \(\dfrac{1}{2} \varphi(60)=8\) ,且经过验证, \(7,11,17,43,53,59,109,181\) 都满足.

解为 \(2\) 和素数 \(60 k+1,60 k+7,60 k+11,60 k+17,60 k+43,60 k+49,60 k+53,60 k+59\)

习题4.2.3

\(p=4 k+1\) 是素数, \(a\)\(k\) 的因子, 证明: \(\left(\dfrac{a}{p}\right)=1\).


证明:由二次互反律 \((\dfrac{a}{p})=(\dfrac{p}{a})(-1)^{\frac{p-1}{2}\frac{a-1}{2}}=(\dfrac{p}{a})=(\dfrac{4k+1}{a})=(\dfrac{4ax+1}{a})=(\dfrac{1}{a})=1\)

习题4.2.6

\((1)\)\(n=2^{m} a+1\), 其中 \(m \geqslant 2,1 \leqslant a<2^{m}\). 如果 \(p\) 为奇素数, 并且 \(\left(\dfrac{n}{p}\right)=-1\), 证明: \(n\) 是素数当且仅当 \(p^{\frac{n-1}{2}} \equiv-1(\bmod n)\).

\((2)\)\(n=2^{m}+1, m \geqslant 2\). 证明: \(n\) 为素数当且仅当 \(3^{\frac{n-1}{2}} \equiv-1\) \((\bmod n)\).


证明:\((1)\)\(n\) 为素数,由于 \(n=2^ma+1\)\(\left(\dfrac{n}{p}\right)=\left(\dfrac{p}{n}\right)=-1\) ,故 \(p^{\frac{n-1}{2}} \equiv-1(\bmod n)\).

另一方面,若 \(p^ \frac{n-1}{2} \equiv-1(\bmod n)\) ,任取 \(n\) 的一个素因子 \(q , q \geq 3\). 令 \(d\)\(p\)\(q\) 的阶,由于

\(p^{\frac{n-1}{2}} \equiv-1(\bmod q)\) ,故 \(p^{2^{m-1} a} \equiv-1(\bmod q)\) ,因此 \(2^{m}\mid d\) ,又 \(d \mid q-1\) ,则 \(2^{m} \mid q-1\)

\(q \geq 2^{m}+1\). 由于 \(n=2^{m} a+1 , a<2^{m}\) ,所以 \(q^2>n\),这样的 \(q\) 只能为 \(n\) 本身,则 \(n\) 为素数

\((2)\)\(\small(1)\) 中取 \(a=1, p=3\)\(n\) 为素数时,满足 \(m\) 为奇数,否则 \(3\)\(n\) 的素因子,此时

\(\left(\dfrac{2^{m}+1}{3}\right)=\left(\dfrac{2}{3}\right)=-1\) ,故满足 \(\small(1)\) 的条件,则 \(3^{\frac{n-1}{2}} \equiv-1\)

\(3^{\frac{n-1}{2}} \equiv-1\),由 \(\small (1)\) 中的推导中没有使用到 \(\left(\dfrac{n}{p}\right)=-1\) 条件知 \(n\) 为素数

习题4.3.2

\(f(x, y)\) 是关于 \(x, y\) 的整系数多项式. \((x, y)=(a, b)\)\((c, d)\) 均是同余方程

\(f(x, y) \equiv 0(\bmod m)\) 的整数解, 即 \(f(a, b) \equiv 0\) \((\bmod m), f(c, d) \equiv 0(\bmod m)\).

称这两组解为模 \(m\) 的同一个解, 是指 \(a \equiv c(\bmod m)\), 并且 \(b \equiv d(\bmod m)\).

\((1)\)\(p\) 为奇素数, \(a\) 为整数. 证明:同余方程 \[ x^{2}-y^{2} \equiv a \quad(\bmod p) \] 的模 \(p\) 解数为 \(p-1\) (若 \(p\not\mid a\)) 或 \(2 p-1\) (若 \(p\mid a\)).

\((2)\) 证明: \[ \sum_{y=0}^{p-1}\left(\frac{y^{2}+a}{p}\right)= \begin{cases}-1, & \text { 若 } p \nmid a, \\ p-1, & \text { 若 } p \mid a .\end{cases} \]


证明:\((1)\)\(p \nmid a\) 时, \((x+y)(x-y) \equiv a(\bmod p)\). 设 \(x+y \equiv b(\bmod p)\) ,其中 \((b, p)=1\).

\(x-y \equiv a b^{-1}(\bmod p)\). 从而 \((x, y)=\left(\dfrac{b+a b^{-1}}{2}, \dfrac{b-a b^{-1}}{2}\right)\). 因此解数为 \(b\) 的个数 \(p-1\).

\(p \mid a\) 时, \(p \mid(x+y)(x-y)\) ,所以 \(x \equiv y(\bmod p)\)\(x \equiv-y(\bmod p)\).

共有 \((0,0),(b,-b),(-b, b)\)\(2\cdotp(p-1)+1=2p-1\) 组解.

\((2)\)\(x^{2} \equiv y^{2}+a(\bmod p) .\left(\dfrac{y^{2}+a}{p}\right)=1\) 时,有两组解, \(\left(\dfrac{y^{2}+a}{p}\right)=-1\) 时,无解

计算其总解数为 \(\sum_{y=0}^{p-1}\left(1+\left(\frac{y^{2}+a}{p}\right)\right)=p+\sum_{y=0}^{p-1}\left(\frac{y^{2}+a}{p}\right)= \begin{cases}p-1, & p \nmid a \\ 2 p-1, & p \mid a\end{cases}\) 因此 \(\sum_{y=0}^{p-1}\left(\frac{y^{2}+a}{p}\right)= \begin{cases}-1, & p \nmid a \\ p-1, & p \mid a\end{cases}\)

习题4.3.3

\(p\) 为奇素数, \(a, b, c \in \mathbf{Z}, p \nmid a, D=b^{2}-4 a c\). 证明: \[ \sum_{x=0}^{p-1}\left(\frac{a x^{2}+b x+c}{p}\right)=\left\{\begin{array}{l} -\left(\dfrac{a}{p}\right), \quad \text { 若 } p \not\mid D, \\ (p-1)\left(\dfrac{a}{p}\right), \text { 若 } p \mid D . \end{array}\right. \]


证明:令 \(y=x+\dfrac{b}{2a}=x+b\cdot 2^{-1}\cdot a^{-1},a^{\prime}=c-b^2\cdot 4^{-1}\cdot a^{-1}\) 代入上题结论得到 \[ \sum_{y=\frac{b}{2a}}^{\frac{b}{2a}+p-1}(\dfrac{ay^2+a'}{p})=\sum_{y=0}^{p-1}(\dfrac{ay^2+a'}{p})=(\dfrac{a}{p})\sum_{y=0}^{p-1}(\dfrac{y^2+a'a^{-1}}{p})=\left\{\begin{array}{l} -\left(\dfrac{a}{p}\right), \quad \text { 若 } p \not\mid D, \\ (p-1)\left(\dfrac{a}{p}\right), \text { 若 } p \mid D . \end{array}\right. \]

习题4.2.4

\(a, b, c \in \mathbf{Z}, p \not\mid a b\), 则同余方程 \[ a x^{2}+b y^{2} \equiv c \quad(\bmod p) \]\(p\) 的解数为 \[ N=\left\{\begin{array}{l} p+(p-1)\left(\dfrac{-a b}{p}\right), \quad \text { 若 } p \mid c, \\ p-\left(\dfrac{-a b}{p}\right), \quad \text { 若 } p \not\mid c . \end{array}\right. \]


证明:\(a x^{2}+b y^{2} \equiv c(\bmod p)\) 等价于 \(x^{2} \equiv-a^{-1} b y^{2}+a^{-1} c(\bmod p) .\) \((a,p)=(b,p)=1\) 解数 \[ \sum_{y=0}^{p-1}\left(1+\left(\frac{-a^{-1} b y^{2}+a^{-1} c}{p}\right)\right)=p+\sum_{y=0}^{p-1}\left(\frac{-a^{-1} b}{p}\right)\left(\frac{y^{2}-c b^{-1}}{p}\right)\\ =p+\left(\frac{-a b}{p}\right) \sum_{y=0}^{p-1}\left(\frac{y^{2}-c b^{-1}}{p}\right)= \begin{cases}p+(p-1)\left(\dfrac{-a b}{p}\right), & p \mid c \\ p-\left(\dfrac{-a b}{p}\right), & p \nmid c\end{cases} \] 其中用到上题结论 \(\displaystyle \sum_{x=0}^{p-1}\left(\frac{a x^{2}+b x+c}{p}\right)=\left\{\begin{array}{l} -\left(\dfrac{a}{p}\right), \quad \text { 若 } p \not\mid D, \\ (p-1)\left(\dfrac{a}{p}\right), \text { 若 } p \mid D . \end{array}\right.\)

\(\large\textcolor{blue}{初等数论第九次作业}\ \ \ \ \ \ _\textcolor{blue}{2022.4.30}\)

习题4.3.1

解下列同余方程:

\((1)\) \(2 x^{2}+3 x+1 \equiv 0 \quad(\bmod 28)\);

\((2)\) \(x^{2} \equiv-1 \quad(\bmod 169)\);

\((3)\) \(x^{2} \equiv 2 \quad(\bmod 98)\);

\((4)\) \(3 x^{2}+x+6 \equiv 0 \quad(\bmod 45)\).


解:\((1)\) \(28 \mid(2 x+1)(x+1)\). 且有 \((2 x+1, x+1)=(1, x+1)=1\) 两者互素,且 \(28=4\times 7\)

\(2 x+1 \equiv 0(\bmod 4), x+1 \equiv 0(\bmod 7)\) 或者 $2 x+1 (), x+1 () $

又或者 \(x+1\equiv 0\pmod{28}\) 解得 \(x \equiv 3,27(\bmod 28)\).

\((2)\) \(x^{2} \equiv-1(\bmod 13)\) 解得 \(x=13 k+5,13 k+8\). 代入 \((13 k+5)^{2} \equiv-1(\bmod 169)\) 可以化简得

\(10 k+2 \equiv 0(\bmod 13)\). 解得 \(k \equiv 5(\bmod 13)\). \((13 k+8)^{2} \equiv-1(\bmod 169)\) 化简得: \(3 k \equiv-5(\bmod 13)\)

解得 \(k \equiv 7(\bmod 13)\). 从而 \(x \equiv 70,99(\bmod 169)\).

\((3)\) \(x^{2} \equiv 2(\bmod 98)\) \(x\) 为偶数. 设 \(x=2 k\) ,则 \(2 k^{2} \equiv 1(\bmod 49)\). 即 \(k^{2} \equiv 25(\bmod 49)\). 考虑

\(k^{2} \equiv 25(\bmod 7)\) ,解得 \(k=7 m+2,7 m+5 .\) 分别代入 \(k^{2} \equiv 25(\bmod 49)\) 解得

\(k \equiv 44,5(\bmod 49)\). 因此 \(x \equiv 10,88(\bmod 98) .\)

\((4)\) \(3 x^{2}+x+6 \equiv 0(\bmod 45)\). 两边模 \(3\)\(x \equiv 0(\bmod 3)\). 令 \(x=3 y\). 代入 \(9 y^{2}+y+2 \equiv 0(\bmod 15) .\)

\(3\) 知, \(y \equiv 1(\bmod 3)\). 设 \(y=3 z+1\) ,代入可知 \(2 z^{2}-z-1 \equiv 0(\bmod 5)\). \((2 z+1)(z-1) \equiv 0(\bmod 5)\).

解得 \(z \equiv 1,2(\bmod 5)\). 从而 \(x \equiv 12,21(\bmod 45)\).

习题5.1.1

求下列方程的全部整数解:

\((1)\) \(2 x^{2}-5 y^{2}=7\); \((2)\) \(x^{2}-2 x y^{2}+5 z^{3}+3=0\);

\((3)\) \(y^{2}=41 x^{3}+3\); \((4)\) \(x^{2}-x y+y^{2}-x-y=0\);

\((5)\) \(y^{2}=x^{3}-6\) \(\tiny\ \) \((6)\) \(y^{2}=x^{3}-x\).


解:\((1)\) \(x^{2} \equiv 0,1,2,4(\bmod 7) .\) 对原方程两边模 \(7\) 知: \(x^{2}+y^{2} \equiv 0(\bmod 7)\). 因此 \(x \equiv y \equiv 0(\bmod 7)\).

两边模 \(49\)\(0\equiv 7\pmod{49}\) 矛盾,从而无解

\((2)\) 方程两边模 \(5\)\(x^{2}-2 x y^{2}+3 \equiv 0(\bmod 5)\) ,故 \(\left(x-y^{2}\right)^{2} \equiv y^{4}-3(\bmod 5)\). 注意到

\(x^{2} \equiv 0,1,-1(\bmod 5),x^{4} \equiv 0,1(\bmod 5)\) ,则 \(y^4\equiv 2,3,4\pmod{5}\) 矛盾,从而无解

\((3)\) 两边模 \(41\) 可知: \(y^{2} \equiv 3(\bmod 41)\). 而 \(\left(\dfrac{3}{41}\right)=\left(\dfrac{41}{3}\right)=\left(\dfrac{-1}{3}\right)=-1\) ,故没有整数解.

\((4)\) \(x^{2}-x y+y^{2}-x-y=0\) 主元后 \(x^2-(y+1)x+y^2-y=0\) 其判别式

\(\Delta=\sqrt{(y+1)^2-4(y^2-y)}=\sqrt{-3y^2+6y+1}\) 为整数,则 \(-3y^2+6y+1=t^2\)

\(3y^2-6y+(t^2-1)=0\) 其判别式 \(\Delta =\sqrt{6^2-4\cdot 3\cdot (t^2-1)}=2\sqrt{3(4-t^2)}\) 也为完全平方数

从而 \(3(4-t^2)\) 为完全平方数,可以得到 \(t^2=1,4\) 解得 \(y=0,1,2\) 回代解得

\((x,y)=(0,0),(0,1),(1,0),(1,2),(2,1),(2,2)\)

\((5)\) 方程两边模 \(8\). 注意到 \(y^{2} \equiv 0,1,4(\bmod 8) , x^{3} \equiv 0, \pm 1, \pm 3(\bmod 8)\) ,对比有 \(x \equiv 7(\bmod 8)\).

\(y^{2}-2=x^{3}-8=(x-2)\left(x^{2}+2 x+4\right)=(x-2)\left((x+1)^{2}+3\right)\). 其中 \((x+1)^{2}+3 \equiv 3(\bmod 8) .\)

任取它的一个素因子 \(p , p\) 为奇数. \(y^{2} \equiv 2(\bmod p)\). 因此 \(1=\left(\dfrac{2}{p}\right)\) ,则 \(p \equiv \pm 1(\bmod 8)\).

这与 \((x+1)^{2}+3 \equiv 3(\bmod 8)\) 矛盾,所以原方程没有整数解.

\((6)\) \(y^{2}=x(x-1)(x+1)\) ,当 \(x=0, \pm 1\) 时, \(y=0\). \(|x| \geq 2\) 时,若 \(x\) 是偶数,\(x, x+1, x-1\) 两两互素.

因此存在整数 \(a, b\) 使得 \(x=a^{2}, x+1=b^{2}\) ,即 \(b^{2}-a^{2}=1\) ,其中 \(b>a>1\). 显然这是无解的.

\(x\) 是奇数,则 \(\left(\dfrac{y}{2}\right)^{2}=x \cdot \dfrac{x+1}{2} \cdot \dfrac{x-1}{2}\). 其中 \(x, \dfrac{x-1}{2}, \dfrac{x+1}{2}\) 两两互素,同理,存在整数 \(a,b\) 使得

\(a^2=\dfrac{x+1}{2},b^2=\dfrac{x-1}{2},a^2-b^2=1\) 故原方程的解为 \((0,0),(1,0),(-1,0)\).

\(\large\textcolor{blue}{初等数论第十次作业}\ \ \ \ \ \ _\textcolor{blue}{2022.5.8}\)

习题5.2.1

求所有正整数 \(m\)\(n\),使 \(2^{m}+3^{n}\) 是完全平方.


解:由题设 \(2^m+3^n=x^2\)\(2,3\not\mid x\) 方程两边模 \(3\)\((-1)^{m}\equiv x^2\equiv 1\pmod{3}\)\(m\equiv 0\pmod{2}\)

\(m\geq 2,2^{m}\geq 4,2^m\equiv 0\pmod{4}\),方程两边模 \(4\)\(0+(-1)^{n}\equiv x^2\pmod{4}\)\(x^2\equiv 0,1\pmod{4}\)

\(n\equiv 0\pmod{2}\)。令 \(m=2k,n=2t\),原方程化为费马方程 \((2^k)^2+(3^t)^2=x^2\) 本原解满足 \[ 2^k=2mn,3^t=m^2-n^2 \]\(m,n\) 一奇一偶,而 \(2^{k-1}=mn\)\(n=1,m=2^{k-1}\) 代入有 \[ 3^t=2^{2(k-1)}-1\Longrightarrow 3^t=(2^{k-1}+1)(2^{k-1}-1) \]\((2^{k-1}-1,2^{k-1}+1)=1\),则 \(2^{k-1}-1=1,k=2,t=1\) 则仅有唯一解 \(m=4,n=2,2^4+3^2=5^2\)

习题5.2.2

求不定方程 \(3^{x}+4^{y}=5^{z}\) 的所有正整数解.


解:方程模 \(3\)\(1\equiv 2^z\equiv (-1)^z\pmod{3}\) 得到 \(z\equiv0\pmod{2}\)。方程模 \(4\)\((-1)^x\equiv 1\pmod{5}\)

\(x\equiv 0\pmod{2}\),令 \(x=2r,z=2k\),原方程化为费马方程 \((3^r)^2+(2^y)^2=(5^k)^2\) 本原解满足 \[ 2^y=2mn,3^r=m^2-n^2=(m-n)(m+n) \]\(2^{y-1}=mn\)\(m=2^s,n=2^t\)。有 \((m+n)-(m-n)=2n=2^{t+1},3\not\mid 2^{t+1}\)\(m-n=1\)

\(2^s=2^t+1\),只有 \(t=0,s=1\) 唯一解,回代得到唯一解 \((x,y,z)=(2,2,2)\)

习题5.2.4

证明: \(x^{4}+4 y^{4}=z^{2}\) 没有正整数解.


证明:假设有解 \((x,y,z)\)\((x,y)>1\) 记素数 \(p\)\((x,y)\) 的一个约数,有 \(p^4\mid x^4+4y^4=z^2\) 得到 \(p^2\mid z\) 从而

\((\dfrac{x}{p},\dfrac{y}{p},\dfrac{z}{p^2})\) 为原方程的整数解,则有限步操作后总能使得 \((x,y)=1=(x^2,y^2)\)\(2\mid x\)\(x=2t\)\(2\not\mid y\)

\(16t^4+4y^4=z^2\Longrightarrow y^4+4t^4=(\dfrac{z}{2})^2\) 与原方程相同,且 \(2\not\mid y\) ,总可以化为 \(2\not\mid x\),有 \((x^2,2y^2)=1\)

\(x^2,2y^2,z\) 为费马方程的本原解。令 \(z\) 为解中最小的,由 \(2\mid 2y^2\) 可知,\(m,n\) 一奇一偶,\((m,n)=1,m>n\) \[ x^2=m^2-n^2,2y^2=2mn,z^2=m^2+n^2 \]\(m^2=x^2+n^2\) 因为 \((m,n)=1\),则 \((x,n)=1\),$(x,n,m) $ 也为费马方程的本原解,由 \(x\) 为奇数 \[ x=c^2-d^2,n=2cd,m=c^2+d^2,y^2=mn=2cd(c^2+d^2) \]\(2 \mid mn=y^2,2\mid y\),则 \(\dfrac{cd(c^2+d^2)}{2}=(\dfrac{y}{2})^2\) 为完全平方数,设 \(c,d\) 中偶数、奇数为 \(s,t\)

\(\dfrac{st(s^2+t^2)}{2}=(\dfrac{y}{2})^2\) 也成立,令 \(s=2k\)\(kt(4k^2+t^2)=(\dfrac{y}{2})^2\)\((c,d)=1=(k,t)\) 三者为完全平方数

\(k=a^2,t=b^2,4k^2+t^2=c^2\) 消元有 \(b^4+4a^4=c^2\)\(c^2=m-d^2<m=\sqrt{z^2-n^2}<z<z^2\)

与假设最小矛盾,从而原方程无解

习题5.2.6

证明: \(1\) 不是同余数, 即不存在面积为 \(1 、\) 三边长为有理数的直角三角形.


证明:原问题等价于满足 \(a^2+b^2=c^2,ab=2d^2\) 的正整数解不存在,假设存在有若 \((a,b)=2\),则

\(2\mid c,4\mid ab=2d^2,2\mid d\),从而 \((\dfrac{a}{2},\dfrac{b}{2},\dfrac{c}{2},\dfrac{d}{2})\) 也为解,若 \((a,b)=p\geq 3\)\(p^2 \mid c^2,d^2\) 同理

\((\dfrac{a}{p},\dfrac{b}{p},\dfrac{c}{p},\dfrac{d}{p})\) 也为解,在有限步之后,总能使得 \((a,b)=1\),满足费马方程,\(a,b\) 一奇一偶,不妨设 \(a=2k\)

\(bk=d^2,(b,k)=1\)\(b=x^2,k=y^2\) 代入 \(a^2+b^2=c^2\)\(4y^4+x^4=c^2\),由上题知该方程无解

习题5.3.1

证明: 形如 \(4^{a}(8 k+7)\) 的正整数不能表为三个整数的平方和.


证明:由 \(x^2\equiv 0,1,4\pmod{8}\) 计算 \(a^2+b^2+c^2\equiv 0,1,2,3,4,5,6\not\equiv 7\pmod{8}\) 从而 \(8k+7\) 的正整数

不能表示为三个整数平方和。对 \(4(8k+7)=a^2+b^2+c^2\)\(x^2\equiv 0,1\pmod{4}\),而三者之和模 \(0\),则

\(a^2\equiv b^2\equiv c^2\pmod{4}\),有 \(a,b,c\equiv 0\pmod{2}\),则可以转化为 \(a'^2+b'^2+c'^2=8k+7\) 的情况

从而 \(4^a(8k+7)\) 均与 \(8k+7\) 等价,从而不能表示为三个整数的平方和

习题5.3.2

\((1)\) 确定哪些整数可表为两个整数的平方差.

\((2)\) 证明: 对任意整数 \(n\), 不定方程 \[ x^{2}+y^{2}-z^{2}=n \] 均有无穷多组正整数解.


解:\((1)\) \(z=x^2-y^2=(x+y)(x-y)\),而 \((x+y)-(x-y)=2y\) 从而 \(z\equiv 0,1,3\pmod{4}\)

而只要 \(z\not\equiv 2\pmod{4}\),分三种情况 \(z=4k=2\cdot 2k,z=4k+1=1\cdot(4k+1),z=4k+3=1\cdot (4k+3)\)

三种情况都对应有 \(x,y\) 解,从而当且仅当 \(z\not\equiv 2\pmod{4}\) 时该整数可以表示为两个整数的平方差

\((2)\)\(y^2-z^2=n-x^2\) 且上问给出只要 \(n-x^2\not \equiv 2\pmod{4}\),就一定有解,而 \(x^2\equiv 0,1\pmod{4}\)

\(n\equiv 0,1,2,3\pmod{4}\),所有情况都有 \(x^2\equiv 0,0,1,0\pmod{4}\) 使得 \(n-x^2\not \equiv 2\pmod{4}\) 成立

而满足上述条件的 \(x\) 有无穷多个,从而对应解 \((x,y,z)\) 也有无穷多个

习题5.3.3

证明: 对任意给定的 \(n \geqslant 1\), 均存在连续 \(n\) 个正整数, 其中每个都不是两个整数的平方和.


证明:取 \(4k+3\) 型素数 \(n\) 个,存在正整数 \(N\) 使得 \[ N=-i+p_{i}\left(\bmod p_{i}^{2}\right) \quad(1 \leqslant i \leqslant n) \] 从而 \(N+i(1\leq i\leq n)\) 分解后 \(p_i\) 的幂次为 \(1\),从而不能写成两个整数平方和,则 \(N+1,\cdots,N+n\) 为所求

\(\large\textcolor{blue}{初等数论第十一次作业}\ \ \ \ \ \ _\textcolor{blue}{2022.5.11}\)

1

\(F\) 是一个域, \(f(x), g(x) \in F[x]\)

\((1)\) 证明存在多项式 \(q(x), r(x) \in F[x]\) 使得 \(f(x)=g(x) q(x)+r(x)\)\(r(x)\) 的次数严格小于 \(g(x)\);

\((2)\)\(h(x)=(f(x), g(x))\) 是首项系数为 1 的最大公因式, 则存在 \(p(x), q(x) \in F(x)\) 使得 \[ f(x) p(x)+g(x) q(x)=h(x) \] \((3)\)\(f(x) \in F_{p}[x]\)\(g(x)\) 是模 \(f(x)\) 的一个缩同余类。则存在 \(h(x)\) 使得 \(g(x) h(x) \equiv 1\) \((\bmod f(x))\)


证明:\((1)\)\(f(x),g(x)\) 最高幂次为 \(x^{n_1},x^{n_2}\),且系数分别为 \(a,b\),不妨设 \(n_1\geq n_2\),由非零元素存在乘法逆元

构造 \(h(x)=ab^{-1}x^{n_1-n_2}\),有 \(ax^{n_1}-bx^{n_2}\cdot ab^{-1}x^{n_1-n_2}=0\),则 \(f(x)-h(x)g(x)\) 首项系数被消去。如此

新的最高幂次不超过 \(n_1-1\),执行上述操作 \(n_1-n_2+1\) 之后,总能得到 \(h_{tot}(x)=\displaystyle \sum_{i=1}^{n_1-n_2+1}h_{i}(x)\)

使得 \(f(x)-g(x)h(x)\) 的最高次数严格小于 \(g(x)\)

\((2)\)\(\small(1)\) 中可知,实际上给出了域上的多项式有带余除法,使用辗转相除法 \[ f(x)=g(x)q_1(x)+f_1(x),g(x)=f_1(x)q_2(x)+g_1(x),\cdots \] 最终得到 \(f_N(x),g_{N-1}(x)=ah(x)\),从而 \(h(x)=a^{-1}f_{N}(x)\),一直回代,每次保证为线性组合

从而有限次数之后总能找到 \(f(x)q(x)+g(x)p(x)=h(x)\)

\((3)\)\(\small(2)\) 可以得到,当 \((f(x),g(x))=1\) 时,存在 \(p(x),h(x)\) 使得 \(f(x)p(x)+g(x)h(x)=1\),从而 \[ g(x)q(x)\equiv 1\pmod{f(x)} \]

2

证明 \(\left(x^{a}-1, x^{b}-1\right)=\left(x^{(a, b)}-1\right)\) 。特别的, \(\left(x^{a^{m}-1}-1, x^{a^{n}-1}-1\right)=x^{a^{(m, n)}}-1\)


证明:不妨设 \(a\geq b\)\((x^a-1,x^b-1)=(x^b-1,x^a-x^b)=(x^b-1,x^b(x^{a-b}-1))\)

\((x^b-1,x^b)=(1,x^b)=1\),则 \((x^a-1,x^b-1)=(x^{a-b}-1,x^b-1)\) 相当于指数项可以相互加减

则类比辗转相除法,经过有限步辗转相除后 \((x^a-1,x^b-1)=(x^{(a,b)}-1,x^{\lambda(a,b)}-1)=x^{(a,b)}-1\)

3

\(f(x) \in F_{p}[x]\)

\((1)\) 证明 \(f(x)^{p}=f\left(x^{p}\right)\);

\((2)\) 证明 \(f(x)\) 的导函数为 \(0\) 当且仅当存在 \(g(x) \in F_{p}[x]\) 使得 \(f(x)=g\left(x^{p}\right)\)


\((1)\) 由费马小定理 \(f(x)^p=\displaystyle (\sum_{i=0}^{n-1}a_ix^i)^{p}=\sum_{i=0}^{n-1}a_i^px^{ip}+pg(x)=\sum_{i=0}^{n-1}a_i^{p-1}a_i(x^{p})^i=\sum_{i=0}^{n-1}a_i(x^p)^{i}=f(x^p)\)

\((2)\)\(f(x)=\displaystyle \sum_{i=0}^{n-1}a_ix^i,f'(x)=\sum_{i=1}^{n-1}ia_ix^{i-1}=0\) 从而存在数论倒数的 \(p\not\mid j\)\(ja_jx^{j-1}=0\Longrightarrow a_j=0\)

从而剩下 \(p\mid i\),使得 \(a_i\) 不一定为 \(0\),则 \(f(x)=\displaystyle \sum_{p\mid i}a_ix^i=g(x^{p})\)

4

证明 \(F:=\left\{a+b i \mid a, b \in \mathbb{Z}_{5}\right\}\) 不是一个域。


证明:假设 \(F\) 为一个域,由域有乘法逆元,考虑 \(1+2i\) 的逆,在 \(F\) 中设为 \(a+bi\),显然 \(F\) 的乘法单位元为 \(1\)

\((1+2i)(a+bi)\equiv 1\pmod{5}\),整理为 \(a\equiv 2b+1\pmod{5},2a\equiv -b\pmod{5}\),得 \(0\equiv 1\pmod{5}\) ,矛盾

\(\large\textcolor{blue}{初等数论第十二次作业}\ \ \ \ \ \ _\textcolor{blue}{2022.5.21}\)

柯召7.22

求出 \(F_{2}\) 上的全部四次不可约多项式.


解:计算 $_{2}(4)=((1)24+(2)22+(4)2^1)=3 $从而共有 \(3\) 个四次不可约多项式, \(F_2\) 上,有

一次不可约多项式为 \(x, x+1\). 有 \(x^2\equiv x\pmod{2}\) 从而当且仅当 \(f(x)\) 的非零系数有偶数个时都不是不可约多项式(验证)

\(F_{2}\) 上的多项式 \(f(x)\)\(x\) 整除当且仅当 \(f(0)=0\) 从而排除掉上述情况有 \(\mathrm{F}_{2}\) 上的不含有一次因子的四次多项式 \[ x^{4}+x^{3}+x^{2}+x+1, x^{4}+x^{3}+1, x^{4}+x^{2}+1, x^{4}+x+1 \]\(x^{4}+x^{2}+1=\left(x^{2}+x+1\right)^{2}\) 可约,则该多项式排除,则 \(F_{2}\)​ 上的全部四次不可约多项式为 \[ x^{4}+x^{3}+x^{2}+x+1, x^{4}+x^{3}+1, x^{4}+x+1 \]

柯召7.23

证明:若 \(p \neq q\) 是两个奇素数, 则 \(F_{p}\) 上首项系数为 \(1\)\(q\) 次不可约多项式的个数为 \(\dfrac{p^{q}-p}{q}\).


证明:由 \(\Phi_{p}(q)=\displaystyle \frac{1}{q} \sum_{l \mid q} \mu(l) p^{\frac{q}{l}}=\dfrac{1}{q}(1\cdot p^q+(-1)\cdot p)=\dfrac{p^q-p}{q}\)

柯召7.27

证明: 若 \(\left(r, p^{n}-1\right)=1\), 则 \(F_{p^{n}}\) 中任一个元 \(\alpha\)\(F_{p^n}\) 中的 \(r\) 次幂元,若 \(r \mid p^n-1\),则 \(\alpha\)\(F_{p^n}\) 中的 \(r\) 次幂元

的充分必要条件是 \(\alpha^{\frac{p^n-1}{r}}=1\)


证明:若 \(\left(r, p^{n}-1\right)=1\) ,则 \(r\) 存在数论倒数 \(r^{-1}\)。 取 \(F_{p}[x]\) 中的一个 \(n\) 次不可约多项式 ,取模 \(f\) 的原根 \(g\).

则对任意的 \(\alpha \in F_{p^{n}}\) ,都存在整数 \(s\) ,使得 \(\alpha \equiv g^{s}(\bmod f)\). 令 \(t=s r^{-1}\) ,则 \(\left(g^{t}\right)^{r} \equiv g^{s} \equiv \alpha(\bmod f)\)

从而 \(\alpha\)\(r\) 次幂元

\(r\mid p^n-1\)\(\Longrightarrow:\) \(\alpha=\beta^r,\alpha^{\frac{p^n-1}{r}}=\beta^{p^n-1}\equiv 1\pmod{f}\) \(\Longleftarrow:\)\(\alpha=g^s\) 代入有 \[ \alpha^{\frac{p^n-1}{r}}=1=g^{\frac{s(p^n-1)}{r}}\equiv 1\pmod{f} \] 由原根的性质可得 \(r\mid s\) 从而 \(\alpha\)\(r\) 次幂元

柯召7.28

证明:设 \(\alpha_{1}, \cdots, \alpha_{p^{n}-1}\)\(F_{p^{n}}\) 中的全体非蕶元, 则 \[ \alpha_{1} \cdots \alpha_{p^{n}-1}=-1 \text {. } \]


证明:类比威尔逊定理的证明,取 \(F_{p}[x]\) 中的一个 \(n\) 次不可约多项式,然后取其原根 \(g\)\[ \alpha_{1} \cdots \alpha_{p^{n}-1} \equiv g^{1+2+\cdots+\left(p^{n}-1\right)} \equiv\left(g^{\frac{p^{n}-1}{2}}\right)^{p^{n}} \equiv(-1)^{p^{n}} \equiv-1(\bmod f) \] 其中 \(g^{\frac{p^n-1}{2}}\equiv -1\pmod{p^n-1}\) 为原根的性质

特殊情况,考虑 \(p=2\)\((-1)^{p^n}\equiv 1\equiv -1\pmod{f}\) 从而对所有素数成立

柯召7.32

证明:若 \(F_{p}\)\(n\) 次不可约多项式的周期为 \(l\), 则 \(p\)\(l\) 的次数为 \(n\).


证明:由原根的性质可知,\(l\mid p^n-1\),令 \(p^x\equiv 1\pmod{l}\) 显然次数 \(r\) 满足 \(r\mid n\)

于是 \(f \mid x^{p^{r}}-x\) ,但 \(f\)\(n\) 次的,故 \(n \leq r\). 于是 \(r=n\) ,即 \(p\)\(l\) 的次数为 \(n\).

\(\large\textcolor{blue}{初等数论第十三次作业}\ \ \ \ \ \ _\textcolor{blue}{2022.5.28}\)

柯召7.34

分别求出 \(F_{2}\) 上多项式 \(f(x)=(x+1)\left(x^{3}+x+1\right)\left(x^{3}+x^{2}+1\right)\) 和 $g(x) = (x^2+x+1)$

\(\left(x^{4}+x+1\right)^{3}\) 的周期.


解:由定理可知周期等于 \([l_1,\cdots,l_n]e,e=\{2^r:r\in Z,r\geq 0,2^r\geq 1(f(x)),3(g(x))\}=1,4\),求 \(l_i\)

\(x+1\mid x-1\) 最小正整数为 \(1\),则对 \(f(x)\)\((x+1)\) 的周期为 \(1\),对 \((x^3+x+1)\)\((x^3+x^2+1)\)

均为不可约多项式,且 \(2^3-1=7\) 为素数,从而两者周期均为 \(7\),故 \(f(x)\) 周期为 \([1,7,7]\cdot 1=7\)

\(x^2-1=3\) 为素数,且 \(x^2+x+1\) 为不可约多项式,则其周期为 \(3\)

\((x^4+x+1)\)\(2^4-1=15\) 为合数,而分别计算 $x5,x31 $ \[ \begin{gathered} x^5=x\cdot x^4\equiv x(-x-1)\equiv -x^2-x\equiv -x(x+1)\not \equiv 1\pmod{x^4+x+1}\\ x^3\not\equiv1\pmod{x^4+x+1} \end{gathered} \]

\(15\)\(x^4+x+1\) 的周期,\(g(x)\) 周期为 \([3,15]\cdot 4=60\),故 \(f(x)\)\(g(x)\) 的周期为 \(7\)\(60\)

柯召7.36

\(F_{2}\) 上多项式 \(x^{2 \cdot 3^{k}}+x^{3^{k}}+1\) 的周期, 并证明它是不可约的。


解:由立方差公式 \(x^{3\cdot 3^{k}}-1=(x^{2 \cdot 3^{k}}+x^{3^{k}}+1)(x^{3^{k}}-1)\)\(x^{2 \cdot 3^{k}}+x^{3^{k}}+1\mid x^{3^{k+1}}-1\) 令其周期为 \(r\)

\(r\mid 3^{k+1}\),而考虑 \(x^{2 \cdot 3^{k}}+x^{3^{k}}+1\mid x^r-1\) 的最高次项,\(r\geq 2\cdot 3^k\),故 \(r=3^{k+1}\)

假设该多项式不可约,则其能被分解为若干个不可约多项式之积,由不可约多项式周期相关定理有 \[ 3^{k+1}=[l_1,l_2,\cdots,l_n]e \]\(e=2^r\)\(e=1,\exists\ s\in[1,n],s.t.l_s=3^{k+1}\) 令该不可约多项式为 \(p(x)\) 令其最高幂次项为 \(n\),有 \[ 3^{k+1}\mid 2^n-1 \] 两边模 \(3\)\((-1)^n\equiv 1\pmod{3}\),则 \(n\equiv 0\pmod{2}\),取 \(n=2t\) 代入有 \(3^{k+1}\mid 4^t-1\)

将其展开 \(4^t-1=(1+3)^t-1\) 其中 \(3\) 的最小幂次为 \(k+1\)\(3^{k+1}\mid 3t,3^k\mid t\)\(n=2t\geq 2\cdot 3^k\)

恰为 \(f(x)\) 的最高幂次,从而该多项式即为其本身,故其不可约

柯召8.16

\(p\) 是一个奇素数, \(d=(m, p-1)\), 证明: \(N\left(x^{m}=a\right)=N\left(x^{d}=a\right)\).


证明:当 \(x^d\equiv a\pmod{p}\) 无解时,\(x^m\equiv a\pmod{p}\) 也无解(否则 \((x^{\frac{m}{d}})^d\) 为前者的解)

又当 \(x^d\equiv a\pmod{p}\) 有解时,其解有 \(d\) 个,设为 \(x_1,\cdots,x_d\) ,又注意到 \((\dfrac{m}{d},p-1)=1\) 则有

\(p-1\) 的完系均为 \(\dfrac{m}{d}\) 次剩余,则 \(x^m\equiv a\pmod{p}\) 等价于 \(x^{\frac{m}{d}}\equiv x_i\pmod{p}\) ,每个方程都只有一解

(取 \(p\) 的原根 \(g\),有 \(\dfrac{m}{d}\mbox{ind}_gx\equiv \mbox{ind}_gx_i\pmod{p-1}\) 则只有一解)故两者解为 \(d\)

柯召8.21

证明:若 \(k \mid p-1, \chi\)\(F_{p}\) 上一个 \(k\) 阶特征, 则 \[ \sum_{j=0}^{k-1} \chi^{j}(-1)=\left\{\begin{array}{l} k, \quad \text { 当 }\left(\dfrac{-1}{p}\right)_{k}=1 \text { 时, } \\ 0 , \quad \text { 当 }\left(\dfrac{-1}{p}\right)_{k}\neq 1 \text { 时. } \end{array}\right. \]


证明:由定理可知,\(\displaystyle \sum_{j=0}^{k-1} \chi^{j}(-1)=N(x^k=-1)\),则当 \((\dfrac{-1}{p})_k=1\) 时,显然该方程有 \(k\) 个解,当 \((\dfrac{-1}{p})_k=1\) 时,该方程无解,故 \(\sum_{j=0}^{k-1} \chi^{j}(-1)=\left\{\begin{array}{l} k, \quad \text { 当 }\left(\dfrac{-1}{p}\right)_{k}=1 \text { 时, } \\ 0 , \quad \text { 当 }\left(\dfrac{-1}{p}\right)_{k}\neq 1 \text { 时. } \end{array}\right.\)

\(\large\textcolor{blue}{初等数论第十四次作业}\ \ \ \ \ \ _\textcolor{blue}{2022.6.2}\)

阶以及雅可比和

\(\chi \neq \chi_{0}\)\(F_{p}\) 上的一个特征。设 \(\rho(a)=\left(\dfrac{a}{p}\right)\) 是唯一的 \(2\) 阶特征。设 \(p \neq 2\)

\((1)\) 证明 \[ \sum_{t \in F_{p}} \chi\left(1-t^{2}\right)=J(\chi, \rho) \] \((2)\)\(0 \neq k \in F_{p}\) 。证明 \[ \sum_{t \in F_{p}} \chi(t(k-t))=\frac{\chi\left(k^{2}\right)}{\chi\left(2^{2}\right)} J(\chi, \rho) \] (提示:利用 \((1)\) )

\((3)\) 证明 \(J(\chi, \chi)=\chi(2)^{-2} J(\chi, \rho)\); (提示:利用 \((2)\))

\((4)\)\(\chi^{2} \neq \chi_{0}\) 。证明 \(g_{1}(\chi)^{2}=\chi(2)^{-2} J(\chi, \rho) g_{1}\left(\chi^{2}\right)\);

\((5)\)\(p \equiv 1(\bmod 3)\)\(\chi\) 是一个 \(3\) 阶特征。证明 \(g_1(\chi)^{3}=p \cdot \chi(2) J(\chi, \rho)\);

\((6)\)\(p \equiv 1(\bmod 4)\)\(\chi\) 是一个 \(4\) 阶特征。证明 \(\chi^{2}=\rho\)\(J(\chi, \chi)=\chi(-1) J(\chi, \rho)\)

(提示: 证明 \(\chi(-1)=(\dfrac{2}{p})\))


解:\((1)\) 由雅可比定义,代入 \(\left(\dfrac{a}{p}\right)=N(x^2=a)-1\)\[ \begin{gathered} J(\chi,\rho)=\displaystyle \sum_{t\in F_p}\chi(1-t)\rho(t)=\sum_{t\in F_p}\chi(1-t)(N(x^2=t)-1)\\ =\sum_{t\in F_p}\chi(1-t)N(x^2=t)-\sum_{t\in F_p}\chi(1-t) =\sum_{t\in F_p}\chi(1-t)N(x^2=t) \end{gathered} \]\(N(x^2=t)\) 对求和有贡献的有两类,当 \(N(x^2=t)=1\)\(t=0\),当 \(N(x^2=t)=2\) 时,\(t\) 为二次剩余 \[ J(\chi,\rho)=\chi(1-0)\cdot 1+2\cdot \sum_{(\frac{t}{p})=1}\chi(1-t) \] 而对每一个二次剩余 \(t=b^2\),且 \(\chi(1-b^2)\)\(\pm b\) 对应同一个 \(t\),且保证所有的 \(b\) 不同,构成 \(F_p\) 中所有非零元素 \[ J(\chi,\rho)=1+2\cdot \sum_{b\in F_p,b\neq 0}\chi(1-b^2)=\sum_{t \in F_{p}} \chi\left(1-t^{2}\right) \] \((2)\)\((2,p)=1\) 以及 \((k,p)=1\),则 \(\{2t\ |\ t\in F_p\}\)\(\{kt\ |\ t\in F_p\}\) 构成 \(F_p\) 中的完系,则 \[ \begin{gathered} \chi(k)^2\sum_{t\in F_p}\chi(1+t)\chi (1-t)=\sum_{t\in F_p}\chi(k+kt)\chi (k-kt)=\chi(k)^2\sum_{t\in F_p}\chi(1-t^2)=\sum_{t\in F_p}\chi(k+t)\chi (k-t)\\=\sum_{t\in F_p}\chi(t)\chi (2k-t) =\sum_{t\in F_p}\chi(2t)\chi (2k-2t)=\chi(2)^2\sum_{t\in F_p}\chi(t)\chi (k-t) \end{gathered} \] 代入 \(\small (1)\) 中结论 \(\displaystyle \sum_{t \in F_{p}} \chi\left(1-t^{2}\right)=J(\chi, \rho)\) 则得到 \(\displaystyle \sum_{t \in F_{p}} \chi(t(k-t))=\frac{\chi\left(k^{2}\right)}{\chi\left(2^{2}\right)} J(\chi, \rho)\)

\((3)\)\(k=1\)\(\displaystyle \sum_{t \in F_{p}} \chi(t(1-t))=\frac{\chi\left(1^2\right)}{\chi\left(2^{2}\right)} J(\chi, \rho)=\sum_{t \in F_{p}}\chi(t)\chi(1-t)=J(\chi,\chi)\)

\(\chi(1)=1\),代入得 \(J(\chi, \chi)=\chi(2)^{-2} J(\chi, \rho)\)

\((4)\) 由雅可比和与高斯和之间的关系有 \(J(\chi,\chi)=\dfrac{g_1(\chi)\cdot g_1(\chi )}{g_1(\chi ^2)}=\chi(2)^{-2} J(\chi, \rho)\)

移项整理得到 \(g_{1}(\chi)^{2}=\chi(2)^{-2} J(\chi, \rho) g_{1}\left(\chi^{2}\right)\)

\((5)\)\(\small(4)\) 等式左右乘以 \(g_1(\chi)\) 可以得到 \(g_{1}(\chi)^{3}=\chi(2)^{-2} J(\chi, \rho) g_{1}\left(\chi^{2}\right)g_1(\chi)\),而由于 \(\chi\)\(3\) 阶特征

\(\chi(2)^3=1\)\(g_{1}(\chi)^{3}=\chi(2) J(\chi, \rho) g_{1}\left(\chi^{2}\right)g_1(\chi)\) 由于 \[ \overline{g(\chi)}=\sum_{t} \overline{\chi(t)} \zeta^{-t}=\chi(-1) \sum_{t} \overline{\chi(-t)} \zeta(-t)=\chi(-1) g(\chi) \]\(\chi(-1)^3=1\)\(\chi(-1)^2=\chi(1)=1\),故 \(\chi(-1)=1\),代入有 \[ g\left(\chi^{2}\right) g(\chi)=g(\bar{\chi}) g(\chi)=\overline{g(\chi)} g(\chi)=|g(\chi)|^{2}=p \]\(g_1(\chi)^{3}=p \cdot \chi(2) J(\chi, \rho)\)

\((6)\) 由于 \(\chi\) 的阶为 \(4\),则 \(\chi^2\) 的阶为 \(2\),而 \(\rho\) 为唯一的二阶特征,故 \(\chi ^2=\rho\),由高斯和与雅可比和之间关系 \[ g(\chi)^{4}=\chi(-1) p J(\chi, \chi) J\left(\chi, \chi^{2}\right)=\chi(-1) p J(\chi, \chi) J(\chi, \rho) \] 又对 \(\small(4)\) 中的等式两边平方得到 \[ g(\chi)^{4}=\chi(2)^{-4} J(\chi, \rho)^{2}\left[g\left(\chi^{2}\right)\right]^{2} \] 而由 \(4\) 阶特征可以得到 \(\chi\left(2^{4}\right)=\chi^{4}(2)=\chi_0(2)=1\), and \(g\left(\chi^{2}\right)=g(\rho)=g\), so \(\left[g\left(\chi^{2}\right)\right]^{2}=g^{2}\) ,而有 \[ g_{a} g_{-a}=(\dfrac{a}{p})(\dfrac{-a}{p}) g^{2}=(\dfrac{-1}{p}) g^{2}\Longrightarrow\sum_{a} g_{a} g_{-a}=\left(\frac{-1}{p}\right)(p-1) g^{2} \] 而使用另一种方法计算左边有 \[ g_{a} g_{-a}=\sum_{x} \sum_{y}\left(\frac{x}{p}\right)\left(\frac{y}{p}\right) \zeta^{a(x-y)}=\sum_{x} \sum_{y}\left(\frac{x}{p}\right)\left(\frac{y}{p}\right) \delta(x, y) p=(p-1) p \] 两式对比有 \(g^{2}=(\dfrac{-1}{p})p\) 由于 \(p\equiv 1\pmod{4}\)\((\dfrac{-1}{p})=1\)\(g^2=p\),则有 \[ \chi(-1) p J(\chi, \chi) J(\chi, \rho)=J(\chi, \rho)^{2} p \] 由于 \(g(\chi)^{4} \neq 0\) 以及 \(|g(\chi)|^{2}=p\),有 \(J(\chi, \rho) \neq 0\), 则两边约去 \(pJ(\chi,\rho)\)\[ \chi(-1) J(\chi, \chi)=J(\chi, \rho) \] \([\chi(-1)]^{2}=\chi\left((-1)^{2}\right)=\chi(1)=1\), 故 \(\chi(-1)=\pm 1\), 从而 \(\chi(-1)^{-1}=\chi(-1)\), 移项得到 \[ J(\chi, \chi)=\chi(-1) J(\chi, \rho) \]

\(\large\textcolor{blue}{初等数论第十五次作业}\ \ \ \ \ \ _\textcolor{blue}{2022.6.12}\)

雅可比和以及加密解密

  1. \(p\) 是一个奇素数, 计算

    \((1)\) \(N\left(x^{3}-y^{3}=1\right), p=37\);

    \((2)\) \(N\left(9 x^{2}+8 y^{2}=a\right), p=17, a \in F_{17}^{*}\)

  2. 使用加密密钥 \(E(x) \equiv x+3(\bmod 26)\) 将信息 “mathematics” 加密.

  3. 已知加密密钥 \(E(x) \equiv x+5(\bmod 26)\) 。将信息 "lttijajsnsl" 解密.

  4. 假设加密密钥 \(E(x) \equiv 3 x+1(\bmod 26)\), 求去密密钥 \(D(y)\), 并将信息 “EBOBO BEBOK" 解密。


1、解:\((1)\) 计算 \(M(x^3+(-y)^3=1)=\displaystyle \sum_{i=0}^2\sum_{j=0}^2\chi ^i(a)\chi^j(1-a)=p+J(\chi,\chi)+J(\chi^2,\chi^2)-2\chi(-1)\)

\(=p-2+J(\chi,\chi)+J(\chi^2,\chi^2)=p-2+J(\chi,\chi)+\overline{J(\chi,\chi)}\)\(J(\chi,\chi)=a+b\omega,\omega=-\dfrac{1}{2}+\dfrac{\sqrt{3}}{2}i\)

得到 \(p-2+2a-b\),由分解定理 \(4p=(2a-b)^2+27(\dfrac{b}{3})^2=148=11^2+27\cdot 1\Longrightarrow a=7,b=3\) 要求 \(2a-b\equiv 1\pmod{3}\)

\(2a-b=-11\) \(N(x^3-y^3=1)=37-2+(-11)=24\)

\((2)\)\(p=17\)\(N(9x^2+8y^2=a)=N(9(x^2-y^2)=a)\)\(\{3x\}\)\(\{x\}\) 均取遍所有 \(F_{p}^*\),故化解为 \(N(x^2-y^2=a)\)

\((\dfrac{-1}{p})=(-1)^{\frac{p-1}{2}}=1\)\(-1\) 为二次剩余,从而 \(-y^2=(a_0)^2(y)^2=(a_0y)^2\) 又可化解为 \(N(x^2+y^2=a)\) \[ \begin{gathered} N(x^2+y^2=a)=\displaystyle \sum_{i=0}^1\sum_{j=0}^1\chi ^i(t)\chi^j(a-t)=p+J(\chi,\chi_0)+J(\chi_0,\chi)+J(\chi,\chi)\\ =p-\chi(-1)=p+\sum_{c\in F_p}\chi(\dfrac{c}{a-c}) \end{gathered} \]\(\dfrac{c}{a-c}=t\Longrightarrow c=\dfrac{at}{t+1}\) 只有 \(t=-1\) 元素没有遍历,从而 \(\displaystyle \sum_{c\in F_p}\chi(\dfrac{c}{a-c})=0-\chi(-1)=-1\)

故原方程有 \(17+(-1)=16\) 个解

2、使用 \(\mbox{python}\) 编程计算,代码如下,加密结果为 \(\mathbf{pdwkhpdwlfv}\)

1
2
3
4
charList = "abcdefghijklmnopqrstuvwxyz"
word = "mathematics"
for i in word:
print(charList[(ord(i) - ord("a") + 3) % 26], end = "")

3、只需改动一行代码,解密结果为 \(\mathbf{goodevening}\)

1
print(charList[(ord(i) - ord("a") - 5) % 26], end = "")

4、反解同余方程得 \(x\equiv 9\cdot (y-1)\equiv 9y+17\pmod{26}\),代码如下,解密结果为 \(\mathbf{BANAN\ \ ABAND}\)

1
print(charList[(9 * (ord(i) - ord("A")) + 17) % 26], end = "

Elementary number theory homework 1-15
https://lr-tsinghua11.github.io/2022/06/17/%E6%95%B0%E5%AD%A6/%E5%88%9D%E7%AD%89%E6%95%B0%E8%AE%BA%E4%BD%9C%E4%B8%9A%E5%90%88%E9%9B%86%EF%BC%881-15%EF%BC%89/
作者
Learning_rate
发布于
2022年6月17日
许可协议